You are on page 1of 80

ERMP 2022 PART I (Unofficial)

1. A 65-year-old man with a history of hypertension, diabetes, and coronary artery


disease has dyspnea, orthopnea, and bilateral lower extremity edema. On
examination, he has elevated jugular venous pressure, bilateral crackles on lung
auscultation, and pitting edema in his lower extremities and his oxygen saturation
is 90% on room air. His chest radiograph shows cardiomegaly and pulmonary
congestion. Which of the following is the most appropriate initial pharmacological
intervention for this patient?
A. Initiate treatment with angiotensin-converting enzyme (ACE) inhibitors
B. Start intravenous furosemide (Lasix.
C. Administer supplemental oxygen
D. Prescribe beta-blockers

The correct answer is B. Start intravenous furosemide (Lasix)


Explanation: This patient has signs and symptoms of acute decompensated heart
failure, which is a medical emergency. Furosemide is a loop diuretic that reduces
the preload and afterload on the heart, and relieves the pulmonary and systemic
congestion.

2. A 45-year-old woman with a history of obesity and type 2 diabetes mellitus


visits her primary care physician for a routine check-up. She has no complaints and
feels well. Her blood pressure at the clinic is 130/80 mmHg. She says that she has
been monitoring her blood pressure at home and it is usually around 160/95 mmHg.
She takes metformin and lisinopril regularly. Her physical examination is
unremarkable. Which of the following is the most likely diagnosis for this patient?
A. White coat hypertension
B. Masked hypertension
C. Resistant hypertension
D. Malignant hypertension
The correct answer is B. Masked hypertension
Explanation:
This condition occurs when a person has a normal blood pressure reading in the
doctor’s office but high blood pressure at home. It can be caused by factors such as
stress, alcohol, caffeine, or smoking at home. Masked hypertension is associated
with increased risk of cardiovascular events and end-organ damage. Therefore, it is
important to confirm the diagnosis with ambulatory blood pressure monitoring or
home blood pressure measurements.

3. A 32-week pregnant woman presents to the emergency department with dyspnea,


orthopnea, and palpitations. She has a history of cardiac illness since childhood
and did not receive any prenatal care. On examination, she has a loud pansystolic
murmur at the apex, crackles in both lung bases, and pitting edema of the lower
extremities. Her blood pressure is 160/100 mm Hg and her pulse is 120 beats/min.
An electrocardiogram shows left ventricular hypertrophy and atrial fibrillation. A
chest x-ray shows cardiomegaly and pulmonary congestion. What is the most
likely diagnosis?
A. Peripartum cardiomyopathy
B. Pulmonary embolism
C. Rheumatic heart disease
D. Hypertensive emergency

The correct answer is C. Rheumatic heart disease


Explanation:
RHD is caused by chronic inflammation of the heart valves due to repeated
episodes of streptococcal infection. It can lead to mitral stenosis, which is the most
common cause of heart failure in pregnant women. The other options are less
likely because they do not explain the history, physical findings, and imaging
results of the patient.
4. A 72-year-old woman with a history of hypertension and hyperlipidemia is
brought to the emergency department by her daughter, who noticed that her mother
complains of palpitations and chest discomfort. On examination, she has an
irregular pulse of 140 beats/min, a blood pressure of 200/110. An
electrocardiogram shows atrial fibrillation with rapid ventricular response. What is
the immediate action indicated for stroke prevention in this patient?
A. Aspirin
B. Clopidogrel
C. Dabigatran
D. Warfarin

Correct Answer: D. Warfarin

5. A 55-year-old man with a history of smoking and hypertension presents to the


emergency department with severe chest pain radiating to his left arm. He is
diaphoretic and nauseous. His vital signs are: blood pressure 180/110 mm Hg,
pulse 110 beats/min, respirations 22 breaths/min, and temperature 37°C. What is
the most appropriate initial diagnostic test for this patient?
A. Electrocardiogram (ECG)
B. Cardiac markers
C. Echocardiogram
D. Coronary angiography

The correct answer is A. Electrocardiogram


Explanation:
This patient has signs and symptoms of acute myocardial infarction, which is a
medical emergency. The first step in the diagnosis of AMI is to obtain an ECG,
which should be done and interpreted with in the first 10 minutes of patient’s
arrival. It can show characteristic changes such as ST-segment elevation, Q waves,
or T wave inversion.
Cardiac markers, such as troponin and creatine kinase-MB, are useful to confirm
the diagnosis of AMI, but they may not be elevated in the early stages of the event.
Echocardiogram can assess the cardiac function and wall motion abnormalities, but
it is not as sensitive or specific as ECG for AMI.
Coronary angiography is the gold standard for identifying the coronary artery
occlusion and determining the revascularization strategy, but it is not the initial test
for AMI⁶.

7. A 45-year-old man with a history of rheumatic heart disease presents to the


emergency department with worsening dyspnea, orthopnea, and paroxysmal
nocturnal dyspnea. He also reports a productive cough with yellowish sputum. He
denies fever, chills, joint pain, or rash. On examination, he has a blood pressure of
160/100 mm Hg, a pulse of 110 beats/min, and a respiratory rate of 28 breaths/min.
He has crackles in both lung bases and a pansystolic murmur at the apex. His
jugular venous pressure is elevated and his lower extremities are edematous. Chest
radiograph shows cardiomegaly and bilateral alveolar infiltrates. What is the most
likely precipitating factor for his acute decompensation?
A. Pneumonia
B. Rheumatic fever
C. Atrial fibrillation
D. Mitral valve stenosis

The correct answer is A. Pneumonia.


Explanation:
This patient has signs and symptoms of acute heart failure due to chronic RHD.
Pneumonia is a common precipitating factor for heart failure exacerbation, as it
causes increased pulmonary congestion, hypoxemia, and systemic inflammation.
Rheumatic fever is unlikely in the absence of fever, joint pain, or rash. Atrial
fibrillation, mitral valve stenosis, and mitral valve regurgitation are complications
of RHD that may contribute to heart failure, but they are not acute triggers.
8. A 12-year-old girl is diagnosed with acute rheumatic fever after having a sore
throat and a positive throat culture for group A streptococcus. She has migratory
polyarthritis, erythema marginatum, and a high-grade fever. She is treated with
penicillin and aspirin and her symptoms improve. Echocardiogram shows mild
mitral regurgitation. What is the most appropriate secondary prevention strategy
for this patient?
A. Benzathine penicillin every 4 weeks for 10 years or until age 21, whichever is
longer
B. Benzathine penicillin every 4 weeks for 5 years or until age 18, whichever is
longer
C. Oral penicillin twice daily for 10 years or until age 21, whichever is longer
D. Oral penicillin twice daily for 5 years or until age 18, whichever is longer

The correct answer is A. Benzathine penicillin every 4wks for 10yrs or until
age 21, whichever is longer.
Explanation:
This patient has ARF with carditis, which puts her at high risk of developing
rheumatic heart disease and recurrent ARF. Secondary prophylaxis with
benzathine penicillin is recommended to prevent streptococcal infections and
reduce the risk of RHD progression. The duration of prophylaxis depends on the
severity of carditis, the age of the patient, and the risk of exposure to streptococcus.
For patients with ARF and carditis, the American Heart Association guidelines
recommend benzathine penicillin every 4 weeks for 10 years or until age 21,
whichever is longer. Oral penicillin is less effective and less reliable than
benzathine penicillin. Aspirin is used for symptomatic relief of arthritis and fever,
but it does not prevent streptococcal infections or RHD.

9. A 65-year-old woman with a history of hypertension and obesity presents to the


emergency department with sudden onset of chest pain and dyspnea. She returned
from a long flight two days ago. On examination, she has a blood pressure of
180/110 mm Hg, a pulse of 120 beats/min, and a respiratory rate of 30 breaths/min.
She is cyanotic and diaphoretic. Her oxygen saturation is 88% on room air. She has
a loud S2 and a right ventricular heave. Her lower extremities are swollen and
tender. Electrocardiogram shows sinus tachycardia, right axis deviation, and
S1Q3T3 pattern. D-dimer is elevated. What is the most appropriate initial
diagnostic test for this patient?
A. Helical CT scan
B. CT angiography
C. Ventilation-perfusion scan
D. Pulmonary angiography

The correct answer is B. CT angiography


Explanation:
This patient has a high clinical probability of pulmonary embolism, based on her
history, symptoms, signs, and laboratory findings. CT angiography is the preferred
diagnostic test for PE, as it has high sensitivity and specificity, and can also
evaluate other causes of chest pain and dyspnea.
Helical CT scan is a non-contrast-enhanced scan that can detect large central PE,
but it has low sensitivity for peripheral PE and cannot exclude alternative
diagnoses.
Ventilation-perfusion scan is an alternative test for patients with contraindications
to contrast or renal impairment, but it may yield indeterminate results in patients
with pre-existing lung disease.
Pulmonary angiography is the gold standard for PE diagnosis, but it is invasive,
expensive, and not widely available.

10. A 3-day-old neonate is admitted to the neonatal intensive care unit for
respiratory distress and poor feeding. He was born at term by vaginal delivery with
no complications. On examination, he has tachypnea, intercostal retractions, and
grunting. His heart rate is 160 beats/min and his blood pressure is 70/40 mm Hg.
He has a normal S1 and S2, with no murmurs or gallops. His chest radiograph
shows cardiomegaly with a cardiothoracic ratio of 0.7. His echocardiogram is
normal, with no structural or functional abnormalities. What is the most likely
explanation for his cardiomegaly on chest radiograph?
A. Inspiratory CXR with poor penetration
B. Congenital heart disease
C. Pericardial effusion
D. Cardiomyopathy

The correct answer is A. Inspiratory CXR with poor penetration


This neonate has cardiomegaly on chest radiograph, but a normal echocardiogram
suggests that the cardiomegaly is an artifact of the chest radiograph, rather than a
true cardiac enlargement. One possible cause of this artifact is an inspiratory CXR
with poor penetration, which can make the heart appear larger than it is. Other
causes of cardiomegaly on CXR, such as congenital heart disease, pericardial
effusion, cardiomyopathy, and sepsis, are unlikely in this case, as they would have
been detected by echocardiogram or other clinical features.

11. A 35-year-old woman presents with fatigue, dry cough, and dyspnea on
exertion. She has no history of smoking or exposure to environmental toxins. A
chest X-ray reveals bilateral hilar lymphadenopathy. A biopsy of one of the lymph
nodes shows noncaseating granulomas. What is the most likely diagnosis?
A. Tuberculosis
B. Silicosis
C. Lymphoma
D. Sarcoidosis

Answer: D. Sarcoidosis
Explanation:
Sarcoidosis is a systemic inflammatory disorder characterized by noncaseating
granulomas in various organs, especially the lungs and lymph nodes. The cause of
sarcoidosis is unknown, but it may involve genetic and environmental factors.
Sarcoidosis typically affects young adults, especially women and African
Americans. The most common symptoms are fatigue, dry cough, and dyspnea, but
some patients may be asymptomatic. Chest X-ray is the initial test of choice, and it
may show bilateral hilar lymphadenopathy, which is a hallmark of sarcoidosis.
Biopsy of an affected organ is required to confirm the diagnosis and rule out other
causes of granulomatous inflammation, such as tuberculosis, lymphoma,
Wegener's granulomatosis, and silicosis.

12. A 65-year-old man with a history of chronic obstructive pulmonary disease and
diabetes mellitus presents with fever, productive cough, and pleuritic chest pain.
On physical examination, he has decreased breath sounds and dullness to
percussion over the left lower lung field. A chest X-ray confirms left lower lobe
consolidation. His blood urea nitrogen (BUN. level is 20 mmol/L). What is the
most appropriate empirical antibiotic therapy for this patient?
A. Ceftriaxone
B. Azithromycin
C. Ceftriaxone plus azithromycin
D. Vancomycin plus piperacillin-tazobactam

Answer: C. Ceftriaxone plus azithromycin


Explanation:
Community-acquired pneumonia (CAP) is an infection of the lower respiratory
tract that occurs in patients who have not been hospitalized or resided in a long-
term care facility within 14 days of the onset of symptoms. Low-risk patients can
be treated as outpatients with oral antibiotics, such as a macrolide (e.g.,
azithromycin) or a doxycycline. Moderate-risk patients may require hospitalization
and intravenous antibiotics, such as a beta-lactam (e.g., ceftriaxone) plus a
macrolide, or a respiratory fluoroquinolone (e.g., levofloxacin) alone. High-risk
patients may need ICU admission and broad-spectrum antibiotics, such as
vancomycin plus piperacillin-tazobactam. In this case, the patient has a CURB-65
score of 2, indicating moderate risk. Therefore, the most appropriate empirical
antibiotic therapy is ceftriaxone plus azithromycin.

13. A 3-year-old boy is brought to the emergency department by his parents with
high fever, drooling, and difficulty breathing. He has a muffled voice and prefers
to sit upright with his neck extended. On physical examination, he has inspiratory
stridor and intercostal retractions. A lateral neck X-ray shows a thumbprint sign in
the hypopharynx. What is the most likely cause of this patient's upper airway
obstruction (UAO)?
A. Epiglottitis
B. Bacterial tracheitis
C. Croup
D. Retropharyngeal abscess

Answer: A. Epiglottitis
Explanation:
Epiglottitis is a severe infection of the epiglottis and surrounding tissues that can
cause life-threatening UAO. It is most commonly caused by Haemophilus
influenzae type b (Hib) in unvaccinated children, but other bacteria, such as
Streptococcus pyogenes, Staphylococcus aureus, and Klebsiella pneumoniae, can
also cause epiglottitis. The typical presentation of epiglottitis is a child with high
fever, drooling, dysphagia, dysphonia, and respiratory distress. The child may
adopt a tripod position, sitting upright with the neck extended and the mouth open,
to maximize the airway. Physical examination may reveal inspiratory stridor,
intercostal retractions, and cyanosis. A lateral neck X-ray may show a thumbprint
sign, which is a swollen epiglottis protruding into the airway
Croup is the most common cause of UAO in children, but it causes a characteristic
barking cough and a steeple sign on X-ray, which is a narrowing of the subglottic
trachea.
Retropharyngeal abscess is a rare complication of bacterial pharyngitis that can
cause UAO, but it usually presents with neck pain, stiffness, and swelling, rather
than drooling and dysphonia.

14. Which of the following drugs has been shown to reduce mortality in
hospitalized patients with COVID-19 who require oxygen therapy or mechanical
ventilation?
A. Hydroxychloroquine
B. Heparin
C. Remdesivir
D. Dexamethasone
E. Azithromycin

Answer: D. Dexamethasone
Explanation:
Dexamethasone is a corticosteroid that has anti-inflammatory and
immunosuppressive effects. It has been found to reduce mortality by 17% in
hospitalized patients with COVID-19 who require oxygen therapy and by 35% in
those who require mechanical ventilation, compared to standard care. Heparin is an
anticoagulant that may prevent thrombotic complications in COVID-19, but its
effect on mortality is unclear. Remdesivir is an antiviral agent that may shorten the
time to recovery in hospitalized patients with COVID-19, but it has not been
shown to reduce mortality. Hydroxychloroquine and azithromycin are not
recommended for the treatment of COVID-19, as they have not been proven to be
effective and may cause serious adverse effects.

15. A 35-year-old man with a history of HIV infection presents to the emergency
department with fever, cough, and dyspnea. His CD4 count is 50 cells/mm3 and he
is not on antiretroviral therapy. His oxygen saturation is 84% on room air. Chest
X-ray shows bilateral interstitial infiltrates. What is the most appropriate initial
management for this patient?
A. Oxygen
B. Prednisolone and cotrimoxazole
C. Trimethoprim-sulfamethoxazole and pentamidine
D. Rifampin, isoniazid, pyrazinamide, and ethambutol
E. Amphotericin B and flucytosine

Answer: A. Oxygen
Explanation:
This patient has signs and symptoms of Pneumocystis jirovecii pneumonia (PCP),
a common opportunistic infection in patients with HIV and low CD4 counts. The
most important initial management for PCP is oxygen supplementation, as
hypoxemia is the main predictor of mortality.
Prednisolone and cotrimoxazole are used as adjunctive therapy to reduce
inflammation and prevent bacterial superinfection, but they are not sufficient to
treat PCP.
Trimethoprim-sulfamethoxazole (TMP-SMX) is the first-line antibiotic for PCP,
and pentamidine is an alternative for patients who are allergic or intolerant to
TMP-SMX.
However, these drugs should be started after oxygen therapy and after ruling out
other causes of pneumonia, such as tuberculosis or fungal infections, which may
require different antibiotics or antifungals.

16. A 25-year-old woman with a history of asthma presents to the clinic with
wheezing, coughing, and chest tightness for the past two days. She says that she
has been using her albuterol inhaler more frequently than usual, but it does not
seem to help much. Her peak expiratory flow rate is 60% of her personal best.
What is the most appropriate pharmacological treatment for this patient?
A. Montelukast
B. Salbutamol
C. Prednisolone
D. Ipratropium

Answer: C. Prednisolone
Explanation:
This patient has an acute exacerbation of asthma, which is characterized by
worsening of asthma symptoms and reduced lung function. The most effective
pharmacological treatment for acute asthma exacerbations is systemic
corticosteroids, such as prednisolone, which reduce airway inflammation and
improve bronchodilation.
Salbutamol is a short-acting beta-agonist (SABA) that provides rapid relief of
bronchospasm, but it does not address the underlying inflammation and may lose
its efficacy with frequent use.
Montelukast is a leukotriene receptor antagonist that is used as a maintenance
therapy for mild to moderate persistent asthma, but it has no role in acute
exacerbations.
Ipratropium is an anticholinergic agent that may be added to SABA for moderate
to severe exacerbations, but it is not as effective as corticosteroids.

17. A 40-year-old woman presents to the clinic with a complaint of excessive thirst
and urination for the past six months. She says that she drinks about 10 liters of
water per day and urinates about 12 times per day. She denies any history of
diabetes, head trauma, or kidney disease. Her serum sodium is 145 mEq/L, serum
osmolality is 295 mOsm/kg, and urine osmolality is 100 mOsm/kg. She is given a
dose of vasopressin and her urine osmolality does not change after one hour. What
is the most likely diagnosis for this patient?
A. Psychogenic polydipsia
B. Nephrogenic diabetes insipidus
C. Central diabetes insipidus
D. Syndrome of inappropriate antidiuretic hormone secretion

Answer: A. Psychogenic polydipsia


Explanation:
This patient has polyuria (urine output > 3 L/day) and polydipsia (excessive thirst.
with low urine osmolality and normal serum osmolality, which are suggestive of
diabetes insipidus (DI), a condition caused by insufficient production or action of
antidiuretic hormone (ADH). However, the lack of response to vasopressin, a
synthetic analog of ADH, rules out both central DI (due to deficient ADH secretion
from the posterior pituitary) and nephrogenic DI (due to impaired ADH action on
the renal tubules). The most likely diagnosis for this patient is psychogenic
polydipsia, a behavioral disorder characterized by compulsive water drinking that
leads to dilutional polyuria and suppression of endogenous ADH. Syndrome of
inappropriate antidiuretic hormone secretion (SIADH) is a condition of excessive
ADH secretion that leads to hyponatremia and high urine osmolality, which are not
present in this patient.

18. A 65-year-old woman with a history of chronic alcoholism is admitted to the


hospital with confusion, weakness, and nausea. Her serum sodium is 120mEq/L,
serum potassium is 3.2mEq/L, serum magnesium is 1.0 mg/dL, and serum calcium
is 8.5 mg/dL. What is the first step in correcting her hyponatremia?
A. Potassium supplementation
B. Magnesium supplementation
C. Sodium chloride infusion
D. Fluid restriction

Answer: B. Magnesium supplementation


Explanation:
This patient has severe hyponatremia, which is defined as serum sodium
<125mEq/L and is associated with neurological symptoms. Hyponatremia can be
caused by various conditions that alter the balance of water and sodium in the body,
such as hypovolemia, hypervolemia, or euvolemia. However, before correcting the
underlying cause of hyponatremia, it is important to check and correct the serum
magnesium level, as magnesium deficiency can impair the renal response to ADH
and cause resistance to treatment. Magnesium supplementation can restore the
normal function of the renal tubules and facilitate the correction of hyponatremia.
Potassium supplementation may also be needed in this patient, as hypokalemia can
also impair the renal response to ADH, but it is not the first step. Sodium chloride
infusion, fluid restriction, and demeclocycline administration are potential
treatments for hyponatremia, depending on the volume status and urine osmolality
of the patient, but they should be done after correcting the magnesium level.
19. A 45-year-old man with a history of chronic renal failure presents to the clinic
with muscle cramps, tingling sensations, and perioral numbness. On physical
examination, he has a positive Trousseau sign, which is elicited by inflating a
blood pressure cuff on the arm and observing carpal spasm. What is the most likely
electrolyte abnormality in this patient?
A. Hyperkalemia
B. Hypercalcemia
C. Hypokalemia
D. Hypocalcemia

Answer: D. Hypocalcemia
Explanation: This patient has signs and symptoms of hypocalcemia, which is
defined as serum calcium < 8.5 mg/dL. Hypocalcemia can cause neuromuscular
irritability, manifesting as muscle cramps, paresthesia, tetany, and seizures.
Trousseau sign is a clinical sign of latent tetany that occurs when the blood flow to
the arm is occluded by a blood pressure cuff, causing ischemia and hypoxia of the
nerves and muscles. Hypocalcemia can be caused by various conditions that affect
the parathyroid glands, the vitamin D metabolism, or the calcium-phosphate
balance. In this patient, chronic renal failure is a likely cause of hypocalcemia, as it
leads to decreased production of active vitamin D, increased phosphate retention,
and reduced calcium absorption. Hypercalcemia, hypokalemia, hyperkalemia, and
hyponatremia are other electrolyte disorders that can cause neuromuscular
symptoms, but they do not cause Trousseau sign.

20. A 3-year-old boy is brought to the pediatrician by his mother, who says that he
has been having bloody diarrhea, vomiting, and fever for the past three days. She
also noticed that he has become pale and lethargic. His blood pressure is 160/100
mmHg, his hemoglobin is 7 g/dL, his platelet count is 50,000/mm3, and his serum
creatinine is 2.5 mg/dL. His peripheral blood smear shows schistocytes. What is
the most likely causative agent of his condition?
A. Escherichia coli
B. Shigella
C. Salmonella
D. Clostridium difficile

Answer: A. Escherichia coli


Explanation:
This patient has hemolytic uremic syndrome (HUS), which is a triad of
microangiopathic hemolytic anemia, thrombocytopenia, and acute kidney injury.
HUS is a type of thrombotic microangiopathy, a group of disorders characterized
by endothelial damage, platelet activation, and formation of microthrombi in the
small vessels. HUS is most commonly caused by infection with Shiga toxin-
producing Escherichia coli (STEC), especially serotype O157:H7, which is
acquired by ingestion of contaminated food or water. Shiga toxin damages the
endothelial cells of the glomeruli and other organs, leading to hemolysis,
thrombocytopenia, and renal failure. Shigella is another bacterium that produces
Shiga toxin, but it is less common than E. coli as a cause of HUS. Salmonella,
Campylobacter, and Clostridium difficile are other causes of infectious diarrhea,
but they do not cause HUS.

21. A 55-year-old man with a history of hypertension and diabetes mellitus


presents to the emergency department with severe diarrhea and vomiting for the
past two days. He says that he has not been able to keep any fluids or food down.
His blood pressure is 90/60 mmHg, his pulse is 110 beats/min, and his temperature
is 37.2°C. His serum creatinine is 2.0 mg/dL, up from 1.2 mg/dL six months ago.
His urine output is 20mL/h. What is the most likely cause of his acute kidney
injury?
A. Acute interstitial nephritis
B. Acute tubular necrosis
C. Prerenal azotemia
D. Acute glomerulonephritis

Answer: C. Prerenal azotemia


Explanation:
This patient has acute kidney injury (AKI), which is defined as a rapid decline in
renal function, resulting in an increase in serum creatinine and a decrease in urine
output. AKI can be classified into three categories: prerenal, intrinsic, and
postrenal, depending on the location of the insult. Prerenal AKI is caused by
decreased renal perfusion due to hypovolemia, hypotension, or decreased cardiac
output. In this patient, severe diarrhea and vomiting have led to volume depletion
and hypotension, which have reduced the blood flow to the kidneys and impaired
their function. Prerenal AKI is usually reversible if the underlying cause is
corrected promptly.
Acute tubular necrosis (ATN) is a type of intrinsic AKI that is caused by damage
to the renal tubular cells due to ischemia or nephrotoxins. ATN typically occurs
after prolonged or severe prerenal AKI, or in the setting of sepsis, rhabdomyolysis,
or contrast-induced nephropathy.
Acute interstitial nephritis (AIN) is another type of intrinsic AKI that is caused by
inflammation of the renal interstitium due to allergic reactions, infections, or drugs.
AIN usually presents with fever, rash, eosinophilia, and urinary eosinophils.
Acute glomerulonephritis (AGN) is also a type of intrinsic AKI that is caused by
inflammation of the glomeruli due to immune-mediated or infectious processes.
AGN usually presents with hematuria, proteinuria, edema, and hypertension.

22. A 2-day-old female neonate is evaluated for poor feeding and lethargy. She
was born at term by vaginal delivery to a healthy mother. Her birth weight was 3.5
kg. Her physical examination is unremarkable. Her serum thyroid-stimulating
hormone (TSH) is 15 mIU/L (normal range: 0.4-4.0 mIU/L) and her serum free
thyroxine (T4) is 1.2 ng/dL (normal range: 0.8-1.8 ng/dL). What is the most likely
diagnosis for this neonate?
A. Primary hypothyroidism
B. Normal variation in neonate
C. Maternal thyroid disease
D. Transient hypothyroxinemia of prematurity
Answer: A. Primary hypothyroidism
Explanation:
This neonate has primary hypothyroidism, which is defined as a deficiency of
thyroid hormone due to a defect in the thyroid gland. Primary hypothyroidism is
the most common cause of congenital hypothyroidism, which affects about 1 in
3000 newborns and is screened for in all neonates. The most common cause of
primary hypothyroidism is thyroid dysgenesis, which is a developmental anomaly
of the thyroid gland, resulting in agenesis, ectopy, or hypoplasia. Other causes
include thyroid dyshormonogenesis, which is a defect in the biosynthesis of
thyroid hormone, or iodine deficiency, which is rare in developed countries.
Primary hypothyroidism is characterized by elevated TSH and low or normal free
T4 levels.
Normal variation in neonate is not a correct answer, as neonates have higher TSH
and lower free T4 levels than adults, but not to the extent seen in this neonate.
Transient hypothyroxinemia of prematurity is a condition of low free T4 and
normal TSH levels in preterm infants, due to immature hypothalamic-pituitary-
thyroid axis and nonthyroidal illness. It is usually asymptomatic and resolves
spontaneously.
Maternal thyroid disease is a cause of transient congenital hypothyroidism, due to
transplacental passage of maternal thyroid-stimulating immunoglobulins (in
Graves' disease) or antithyroid drugs (in Hashimoto's thyroiditis). It usually
presents with low TSH and high free T4 levels, and it resolves after the maternal
antibodies or drugs are cleared from the neonate's circulation.

23. A 35-year-old woman presents to the clinic with complaints of palpitations,


weight loss, and heat intolerance for the past three months. She says that she has
been feeling restless and anxious, and that she has difficulty sleeping. Her physical
examination reveals a resting heart rate of 120 beats/min, a blood pressure of
150/90 mmHg, and a fine tremor of the hands. Her thyroid gland is enlarged and
diffusely tender. Her serum thyroid-stimulating hormone (TSH) is 0.01 mIU/L
(normal range: 0.4-4.0 mIU/L). What is the best parameter to confirm the diagnosis
of hyperthyroidism in this patient?
A. Free T4
B. Free T3
C. Total T4
D. Total T3

Answer: A. Free T4
Explanation:
This patient has signs and symptoms of hyperthyroidism, which is defined as an
excess of thyroid hormone in the body. Hyperthyroidism can be caused by various
conditions that stimulate the synthesis and secretion of thyroid hormone, such as
Graves' disease, toxic multinodular goiter, or thyroiditis. Hyperthyroidism is
characterized by suppressed TSH and elevated free T4 and/or free T3 levels. The
best parameter to confirm the diagnosis of hyperthyroidism is free T4, as it reflects
the biologically active fraction of thyroid hormone that is not bound to serum
proteins. Free T4 is more sensitive and specific than total T4, which can be
affected by changes in thyroid-binding globulin (TBG) levels.
Free T3 is also a useful parameter to diagnose hyperthyroidism, especially in cases
of T3 toxicosis, where free T4 levels may be normal. However, free T3 is less
reliable than free T4, as it can be influenced by nonthyroidal illness, drugs, or
nutritional factors.
Total T3 is also less accurate than free T4, as it can be affected by TBG levels and
other factors. Thyroid peroxidase antibodies are markers of autoimmune thyroid
disease, such as Graves' disease or Hashimoto's thyroiditis, but they are not
diagnostic of hyperthyroidism, as they can be present in euthyroid or hypothyroid
patients as well.

24. A 50-year-old man with a history of peptic ulcer disease presents to the
emergency department with severe epigastric pain, nausea, and vomiting for the
past two days. He says that he has been taking nonsteroidal anti-inflammatory
drugs (NSAIDs. for his chronic back pain. His physical examination reveals a
distended abdomen and a succussion splash. An upper gastrointestinal series shows
a large gastric outlet obstruction (GOO. due to a duodenal ulcer. What is the most
likely acid-base and electrolyte disturbance in this patient?
A. Hyperchloremic hyperkalemic metabolic acidosis
B. Hypochloremic hypokalemic metabolic acidosis
C. Hyperchloremic hyperkalemic metabolic alkalosis
D. Hypochloremic hypokalemic metabolic alkalosis

Answer: D. Hypochloremic hypokalemic metabolic alkalosis

Explanation: This patient has a gastric outlet obstruction (GOO), which is a


mechanical obstruction of the pylorus or the duodenum, preventing the passage of
gastric contents into the small intestine. GOO can be caused by various conditions,
such as peptic ulcer disease, gastric cancer, or Crohn's disease. GOO can lead to
acid-base and electrolyte disturbances, such as hypochloremic hypokalemic
metabolic alkalosis, due to the loss of gastric acid and fluids through vomiting.
Gastric acid contains hydrochloric acid (HCl) and potassium (K+., so vomiting
leads to a decrease in serum chloride (Cl-) and potassium levels. The loss of
hydrogen ions (H+) causes a rise in serum bicarbonate (HCO3-) and pH,

25. A 45-year-old man with a history of peptic ulcer disease underwent a


successful H.pylori eradication therapy six months ago. He has no symptoms of
dyspepsia or abdominal pain. What is the most appropriate next step in his
management?
A. Repeat endoscopy with biopsy
B. Repeat urea breath test
C. Discontinue proton pump inhibitors
D. Start prophylactic antibiotics

Answer: B. Repeat urea breath test


Explanation:
A non-invasive test, such as a urea breath test or a stool antigen test, is
recommended to confirm the eradication of H.pylori after treatment. Endoscopy
with biopsy is invasive and costly, and not indicated in asymptomatic patients.
Proton pump inhibitors may interfere with the accuracy of the urea breath test, and
should be discontinued for at least two weeks before testing. Antibiotics are not
needed for prophylaxis, and may increase the risk of resistance.

26. A 55-year-old woman with a history of chronic NSAID use presented with
acute onset of severe epigastric pain and peritonitis. She was diagnosed with a
perforated peptic ulcer and underwent an emergency Graham's patch repair. She
was started on intravenous antibiotics and proton pump inhibitors. What is the
most appropriate follow-up strategy for this patient?
A. H.pylori testing and treatment
B. Highly selective vagotomy
C. Repeat endoscopy in 6-8 weeks
D. Discontinue NSAIDS and start misoprostol

Answer: C. Repeat endoscopy in 6-8 weeks


Explanation:
A Graham's patch repair is a surgical technique that involves suturing an omental
patch over the perforation site. It is a simple and effective method of managing
perforated peptic ulcers, but it does not address the underlying cause of the ulcer.
Therefore, a repeat endoscopy is recommended in 6-8 weeks to assess the healing
of the ulcer and to exclude malignancy.
Highly selective vagotomy is a procedure that involves cutting the vagal nerve
fibers that stimulate acid secretion, but it is rarely performed nowadays due to the
availability of effective medical therapy.
H.pylori testing and treatment is indicated for patients with peptic ulcer disease,
but it is not urgent in the setting of perforation.
NSAIDs should be discontinued if possible, and misoprostol may be used as a
prophylactic agent in high-risk patients, but these measures are not sufficient to
prevent recurrence of ulcers.
27. A 60-year-old woman with a history of hypertension and dyslipidemia has been
taking low-dose aspirin daily for cardiovascular risk reduction. She complains of
epigastric pain and heartburn that worsen after meals. She has no history of peptic
ulcer disease or H. pylori infection. She takes no other medications. What is the
best management strategy for her condition?
A. Stop aspirin and start omeprazole
B. Continue aspirin and start omeprazole
C. Stop aspirin and test for h. Pylori
D. Continue aspirin and test for H. pylori

Answer: B. Continue aspirin and start omeprazole

Explanation:
The patient has symptoms of gastroesophageal reflux disease (GERD., which is a
common side effect of aspirin use. Aspirin inhibits the synthesis of prostaglandins,
which protect the gastric mucosa from acid and pepsin. Stopping aspirin may
relieve her symptoms, but it would also increase her risk of cardiovascular events,
such as myocardial infarction or stroke. Therefore, the best option is to continue
aspirin and start omeprazole, which is a proton pump inhibitor that reduces gastric
acid secretion and heals esophagitis. Option A is not advisable because it exposes
the patient to unnecessary cardiovascular risk. Option C is not indicated because
the patient has no history of peptic ulcer disease or H. pylori infection, which are
the main causes of gastric ulcers. Option D is not necessary because testing for H.
pylori would not change the management of GERD.

28. A 65-year-old patient with abdominal pain of one month duration and blood
mixed stool. Stool examination is non-revealing. What is the most appropriate next
step in the evaluation of this patient?
A. Stool culture
B. Colonoscopy
C. Abdominal ultrasound
D. Fecal immunochemical test

Answer: B. Colonoscopy
Explanation:
The patient’s age, abdominal pain, and blood mixed stool are suggestive of
colorectal cancer, which is the second leading cause of cancer-related deaths in the
United States. The stool examination is non-revealing, as it does not detect occult
blood, parasites, or pathogens. The most appropriate next step is to perform a
colonoscopy, which is the gold standard for the diagnosis and screening of
colorectal cancer. It allows direct visualization of the colon and rectum, and biopsy
of any suspicious lesions.
A fecal immunochemical test is a non-invasive screening test, but it has a low
sensitivity and specificity, and requires confirmation by colonoscopy if positive.

29. A 4-year-old boy is brought to the emergency department by his parents, who
report that he has been vomiting, having abdominal pain, and acting lethargic for
the past two days. They also mention that he has a history of sickle cell disease,
and has been taking hydroxyurea and folic acid regularly. On physical examination,
the boy is pale, dehydrated, and tachycardic. His abdomen is distended and tender,
especially in the epigastric region. His laboratory tests show a hemoglobin level of
7 g/dL, a white blood cell count of 15,000/mm3, and a serum glucose level of 250
mg/dL. What is the most likely diagnosis?
A. Acute pancreatitis
B. Acute hepatitis
C. Viral gastroenteritis
D. Sickle cell crisis

Answer: A. Acute pancreatitis


Explanation:
The boy's history and presentation are suggestive of acute pancreatitis, which is a
rare but serious complication of sickle cell disease. The sickling of red blood cells
can cause microvascular occlusion and ischemia of the pancreas, leading to
inflammation and necrosis. The clinical features of acute pancreatitis include
vomiting, abdominal pain, and elevated serum amylase and lipase levels. The boy's
serum glucose level is also high, which may be due to impaired insulin secretion
by the damaged pancreatic beta cells.
Sickle cell crisis may cause abdominal pain and anemia, but it does not explain the
boy's pancreatic involvement or hyperglycemia.

30. A 35-year-old woman with a history of ulcerative colitis for 10 years presents
with pruritus, fatigue, and jaundice. She has no history of alcohol use, drug abuse,
or blood transfusions. On physical examination, she has scleral icterus,
hepatomegaly, and mild ascites. Her laboratory tests show a serum bilirubin level
of 6 mg/dL, an alkaline phosphatase level of 500 U/L, and a positive
antimitochondrial antibody test. What is the most likely diagnosis?
A. Primary sclerosing cholangitis
B. Primary biliary cirrhosis
C. Autoimmune hepatitis
D. Hepatocellular carcinoma

Answer: A. Primary sclerosing cholangitis


Explanation:
The woman's history and presentation are suggestive of primary sclerosing
cholangitis (PSC), which is a chronic inflammatory disorder of the bile ducts,
characterized by fibrosis, strictures, and cholestasis. PSC is strongly associated
with inflammatory bowel disease, especially ulcerative colitis. The clinical features
of PSC include pruritus, fatigue, jaundice, and hepatosplenomegaly. The laboratory
findings of PSC include elevated serum bilirubin and alkaline phosphatase levels,
and positive antinuclear antibody and anti-smooth muscle antibody tests. The
antimitochondrial antibody test is positive in primary biliary cirrhosis (PBC),
which is another chronic cholestatic disorder, but it affects mainly the intrahepatic
bile ducts and is more common in middle-aged women without inflammatory
bowel disease.
Autoimmune hepatitis is a chronic inflammatory disorder of the liver,
characterized by elevated serum transaminase levels, hypergammaglobulinemia,
and positive autoantibodies, such as anti-liver kidney microsomal antibody and
anti-liver cytosol antibody.
Hepatocellular carcinoma is a malignant tumor of the liver, which can cause
weight loss, abdominal pain, and ascites. It is usually associated with chronic viral
hepatitis, cirrhosis, or aflatoxin exposure.

31. A 50-year-old man with a history of alcohol abuse presents with abdominal
distension and lower extremity edema. He has spider angiomas on his chest and
palmar erythema. A paracentesis is performed and the fluid is sent for analysis.
The serum-ascites albumin gradient (SAAG. is 1.4 g/dL) Which of the following is
the most likely diagnosis?
A. Cirrhosis
B. Tuberculosis
C. Nephrotic syndrome
D. Pancreatic cancer

The correct answer is A. Cirrhosis


Explanation:
Transudative ascites is defined by a SAAG of ≥ 1.1 g/dL and is typically caused by
portal hypertension, which can result from cirrhosis. The other options are more
likely to cause exudative ascites, which has a SAAG of < 1.1 g/dL and is due to
inflammation, infection, or malignancy.

32. A 40-year-old woman with a history of autoimmune hepatitis presents with


fatigue, pruritus, and jaundice. She has not been compliant with her
immunosuppressive therapy. Laboratory tests show elevated liver enzymes and
bilirubin. A liver biopsy is performed and shows bridging fibrosis, interface
hepatitis, and plasma cell infiltration. Which of the following is the most
appropriate treatment for this patient?
A. Lamivudine and adefovir
B. Ursodeoxycholic acid and cholestyramine
C. Pegylated interferon and ribavirin
D. Liver transplantation

The correct answer is D. Liver transplantation


Explanation:
This patient has advanced autoimmune hepatitis, which is characterized by chronic
inflammation of the liver, autoantibodies, and hypergammaglobulinemia. The
histologic features of bridging fibrosis and interface hepatitis indicate severe liver
damage and poor prognosis. The standard treatment for autoimmune hepatitis is
prednisone and azathioprine, but this patient has failed to respond to this regimen
and has developed decompensated liver disease. Therefore, the only definitive
treatment option is liver transplantation.

33. A 25-year-old woman who is 28 weeks pregnant presents for a routine prenatal
visit. She is asymptomatic and has no significant medical history. She is screened
for hepatitis B and is found to be positive for hepatitis B surface antigen (HBsAg)
and hepatitis B e antigen (HBeAg.. Her serum hepatitis B virus (HBV) DNA level
is 2000 IU/mL. Which of the following is the most appropriate management for
this patient?
A. Start tenofovir disoproxil fumarate (TDF). prophylaxis
B. Start long-term antiviral treatment
C. Administer hepatitis B immune globulin (HBIG) and HBV vaccine to the
newborn
D. Perform cesarean section delivery
The correct answer is A. Start tenofovir disoproxil fumarate (TDF)
prophylaxis.
Explanation:
This patient has chronic hepatitis B infection, which is defined by the presence of
HBsAg for more than 6 months. She is also highly infectious, as indicated by the
positive HBeAg and the detectable HBV DNA level. The risk of mother-to-child
transmission (MTCT) of HBV is high, especially if the HBV DNA level is >
200,000 IU/mL. The recommended strategy to prevent MTCT is to administer
HBIG and HBV vaccine to the newborn within 12 hours of birth, and to start
antiviral prophylaxis with TDF for the mother in the third trimester if the HBV
DNA level is > 200 IU/mL. Long-term antiviral treatment is not indicated for this
patient, as she is asymptomatic and has normal liver function.
Cesarean section delivery does not reduce the risk of MTCT.

34. A 60-year-old man presents with a painless bulge in his left groin that he
noticed a few months ago. He says that the bulge becomes more prominent when
he coughs or strains, and that he can sometimes push it back into his abdomen. He
has no history of abdominal surgery or trauma. On physical examination, the bulge
is located below the inguinal ligament and medial to the femoral pulse. It does not
extend into the scrotum. The bulge increases in size with the Valsalva maneuver.
Which of the following is the most likely diagnosis?
A. Direct inguinal hernia
B. Indirect inguinal hernia
C. Femoral hernia
D. Obturator hernia

The correct answer is C. Femoral hernia


Explanation:
This patient has a femoral hernia, which is a protrusion of abdominal contents
through the femoral ring, below the inguinal ligament and medial to the femoral
vessels. Femoral hernias are more common in women, but can occur in men as
well. They present as painless groin masses that increase in size with increased
intra-abdominal pressure. They have a high risk of incarceration and strangulation,
and require surgical repair. The other options are less likely, as they have different
anatomical locations and characteristics.

35. A 55-year-old man with a history of gastroesophageal reflux disease (GERD.


presents with dysphagia and weight loss. He has been taking over-the-counter
antacids for years, but his symptoms have not improved. He denies smoking or
alcohol use. An upper endoscopy is performed and shows a 3-cm ulcerated mass in
the distal esophagus. Biopsy of the mass reveals columnar epithelium with goblet
cells and atypical nuclei. Which of the following is the most likely diagnosis?
A. Metaplasia
B. Dysplasia
C. Adenocarcinoma
D. Squamous cell carcinoma

The correct answer is C. Adenocarcinoma


Explanation:
This patient has esophageal adenocarcinoma, which is a malignant tumor arising
from the glandular cells of the esophagus. The main risk factor for esophageal
adenocarcinoma is chronic GERD, which causes damage to the squamous
epithelium of the lower esophagus and its replacement by columnar epithelium
with goblet cells, a condition known as Barrett esophagus. Barrett esophagus is a
type of metaplasia, which is a reversible change of one cell type to another in
response to chronic irritation. However, Barrett esophagus can progress to
dysplasia, which is a precancerous lesion characterized by abnormal cell growth
and differentiation. Dysplasia can then evolve to adenocarcinoma, which is an
invasive malignancy with atypical nuclei and loss of normal tissue architecture.
Therefore, the biopsy findings of this patient are consistent with adenocarcinoma,
not metaplasia or dysplasia. Squamous cell carcinoma is another type of
esophageal cancer, but it arises from the squamous epithelium and is associated
with smoking, alcohol, and hot beverages.
36. A 25-year-old woman undergoes an appendectomy for perforated appendicitis.
On postoperative day2, she develops erythema and pain at the surgical site, and a
small amount of serosanguineous fluid drains from the wound. Her temperature is
38.2°C (100.8°F) and her pulse is 110/min. Which of the following is the most
appropriate management for this patient?
A. Continue current management and monitor wound healing
B. Explore the wound and debride any necrotic tissue
C. Perform a laparotomy and inspect the abdominal cavity
D. Start broad-spectrum antibiotics and change the dressing daily
E. Refer the patient to a plastic surgeon for wound closure

The correct answer is B. Explore the wound and debride any necrotic tissue.
Explanation:
This patient has signs of wound infection, which is a common complication of
appendectomy, especially in cases of perforation. The best treatment for wound
infection is wound exploration, irrigation, and debridement of any devitalized
tissue, followed by packing or leaving the wound open to heal by secondary
intention. Antibiotics are usually not indicated unless there is evidence of systemic
infection or cellulitis. Laparotomy is not necessary unless there is suspicion of
intra-abdominal abscess or peritonitis. Plastic surgery referral is not required for
wound closure, as most wounds will heal with conservative management.

37. A 70-year-old woman is admitted to the hospital with severe abdominal pain,
distension, and obstipation for the past 3 days. She has a history of hypertension
and diabetes mellitus. On physical examination, she has a tympanic abdomen with
decreased bowel sounds and diffuse tenderness. An abdominal x-ray is obtained
and shows the following image:
Which of the following is the most appropriate initial management for this patient?
A. Rectal tube insertion
B. Nasogastric tube insertion
C. Laparotomy
D. Barium enema

The correct answer is A. Rectal tube insertion


Explanation:
This patient has signs and symptoms of sigmoid volvulus, which is a twisting of
the sigmoid colon around its mesentery, causing bowel obstruction. The abdominal
x-ray shows a characteristic "coffee bean" sign, which is a dilated loop of colon
with its apex pointing to the right upper quadrant. The initial management of
sigmoid volvulus is rectal tube insertion, which can decompress the bowel and
untwist the volvulus in up to 80% of cases. Nasogastric tube insertion is not helpful,
as it does not reach the site of obstruction. Colonoscopy can be used as a second-
line option if rectal tube insertion fails, or as a diagnostic tool if the diagnosis is
uncertain. Barium enema is contraindicated, as it can worsen the obstruction and
increase the risk of perforation. Laparotomy is reserved for cases with signs of
ischemia, necrosis, or perforation, or for recurrent or refractory volvulus.

38. A 40-year-old woman presents with chronic right lower quadrant pain, diarrhea,
and weight loss. She has a history of Crohn disease and has been taking
mesalamine for maintenance therapy. On physical examination, she has a palpable
mass in the right iliac fossa. An abdominal CT scan shows a thickened terminal
ileum with adjacent fat stranding and enlarged lymph nodes. Which of the
following is the most likely diagnosis?
A. Appendiceal abscess
B. Crohn disease complication
C. Ileocecal tuberculosis
D. Psoas abscess

The correct answer is B. Crohn disease complication


Explanation:
This patient has signs and symptoms of a flare-up of Crohn disease, which is a
chronic inflammatory bowel disease that can affect any part of the gastrointestinal
tract, but most commonly involves the terminal ileum. Crohn disease can cause
various complications, such as strictures, fistulas, abscesses, and masses, which
may require surgical intervention.

39. A 60-year-old man presents with occult blood in his stool, detected on a routine
screening test. He has no other symptoms and no significant medical history. A
colonoscopy is performed and reveals a polypoid mass in the right hepatic flexure,
which is biopsied and confirmed to be adenocarcinoma. Which of the following is
the most appropriate surgical management for this patient?
A. Right hemicolectomy
B. Right colectomy
C. Segmental resection of the hepatic flexure
D. Transverse colectomy

The correct answer is A. Right hemicolectomy


Explanation:
This patient has right-sided colon cancer, which is a malignant tumor arising from
the glandular cells of the colon. The standard surgical treatment for right-sided
colon cancer is right hemicolectomy, which involves the removal of the cecum,
ascending colon, hepatic flexure, and proximal transverse colon, along with the
associated lymph nodes and blood vessels. This provides adequate margins and
lymphadenectomy, and reduces the risk of recurrence and metastasis.

40. A 50-year-old woman presents with fatigue, pallor, and glossitis. She has a
history of pernicious anemia and has been taking oral vitamin B12 supplements for
the past year. Laboratory tests show a macrocytic anemia with a mean corpuscular
volume (MCV) of 110 fL, a low serum vitamin B12 level, and a high serum
homocysteine level. A peripheral blood smear shows oval macrocytes and hyper
segmented neutrophils. Which of the following is the most likely diagnosis?
A. Pernicious anemia
B. Tropical sprue
C. Celiac disease
D. Folate deficiency

The correct answer is A. Pernicious anemia


Explanation:
This patient has signs and symptoms of vitamin B12 deficiency, which causes a
megaloblastic anemia and neurological dysfunction. The most common cause of
vitamin B12 deficiency is pernicious anemia, which is an autoimmune disorder
that impairs the absorption of vitamin B12 by destroying the gastric parietal cells
that produce intrinsic factor. Oral vitamin B12 supplements are not effective in
pernicious anemia, as they require intrinsic factor for absorption. The treatment of
choice is intramuscular vitamin B12 injections.

41. A 25-year-old man presents with fatigue, fever, and bleeding gums. He has a
history of recurrent infections and easy bruising. A complete blood count shows
leukocytosis with 80% blasts. A peripheral blood smear shows auer bodies (image
below)

Which of the following is the most likely diagnosis?


A. Acute myeloid leukemia
B. Chronic myeloid leukemia
C. Chronic lymphocytic leukemia
D. Acute lymphoblastic leukemia

The correct answer is A. Acute myeloid leukemia


Explanation:
This patient has signs and symptoms of acute leukemia, which is a malignant
proliferation of immature hematopoietic cells. The peripheral blood smear shows
Auer rods, which are cytoplasmic inclusions composed of fused granules, and are
pathognomonic for acute myeloid leukemia.
42. A 65-year-old man presents with urinary hesitancy, frequency, and nocturia.
He has a history of benign prostatic hyperplasia and takes tamsulosin. On digital
rectal examination, he has an enlarged prostate with obliteration of the median
sulcus. Which of the following is the best initial investigation for this patient?
A. Prostate biopsy
B. Transrectal ultrasound
C. Serum prostate-specific antigen
D. Cystoscopy

The correct answer is C. Serum prostate-specific antigen


Explanation:
This patient has signs and symptoms of bladder outlet obstruction due to prostatic
enlargement, which can be caused by benign or malignant processes. The best
initial investigation for this patient is serum prostate-specific antigen (PSA), which
is a marker of prostatic tissue and can be elevated in both benign and malignant
conditions. A high or rising PSA level may indicate the need for further evaluation
with prostate biopsy or imaging.

43. A 5-year-old boy presents with heavy bleeding after circumcision. He has a
history of prolonged bleeding after minor injuries and dental procedures. His
maternal uncle and grandfather also had similar bleeding problems. Laboratory
tests show normal platelet count, bleeding time, prothrombin time, and activated
partial thromboplastin time. Which of the following is the best management for
this patient?
A. Recombinant factor VIII
B. Plasma-derived factor VIII
C. Fresh frozen plasma
D. Tranexamic acid
The correct answer is A. Recombinant factor VIII
Explanation:
This patient has signs and symptoms of hemophilia A, which is an X-linked
recessive disorder caused by a deficiency of factor VIII. Hemophilia A causes a
bleeding diathesis with spontaneous or excessive bleeding after trauma or surgery,
especially into joints and muscles. The best management for hemophilia A is
replacement therapy with recombinant factor VIII, which is safer and more
effective than plasma-derived factor VIII.
Plasma-derived factor VIII carries a risk of viral transmission and inhibitor
formation. Fresh frozen plasma contains factor VIII, but in low concentrations and
large volumes, and is not recommended for hemophilia A.
Tranexamic acid is an antifibrinolytic agent that can be used as an adjunctive
therapy, but not as a primary treatment for hemophilia A.

44. A 32-year-old woman who is 28 weeks pregnant presents with swelling and
pain in her left leg. She has no history of trauma or surgery. On physical
examination, she has a positive Homan sign and a palpable cord in the left
popliteal fossa. A Doppler ultrasound confirms the diagnosis of deep venous
thrombosis. Which of the following is the best management for this patient?
A. Unfractionated heparin
B. Warfarin
C. Low-molecular-weight heparin
D. Rivaroxaban

The correct answer is C. Low-molecular-weight heparin


Explanation:
This patient has deep venous thrombosis (DVT), which is a blood clot in the deep
veins, usually of the lower extremities. Pregnancy is a risk factor for DVT, due to
increased venous stasis, hypercoagulability, and compression of the inferior vena
cava by the gravid uterus. The best management for DVT in pregnancy is
anticoagulation with low-molecular-weight heparin (LMWH), which is safe and
effective for both the mother and the fetus.
Unfractionated heparin can also be used, but it requires more frequent monitoring
and has more side effects than LMWH.
Warfarin is contraindicated in pregnancy, as it can cross the placenta and cause
fetal bleeding or malformations.
Rivaroxaban is a direct oral anticoagulant that has not been adequately studied in
pregnancy and is not recommended.

45. A 40-year-old man presents with a painless lump in his neck that he noticed a
few weeks ago. He has no history of fever, night sweats, weight loss, or infection.
He smokes half a pack of cigarettes per day and drinks occasionally. On physical
examination, he has a 3-cm firm, non-tender, and mobile lymph node in the left
cervical region. There are no other palpable lymph nodes or organomegaly. Which
of the following is the best initial investigation for this patient?
A. Fine-needle aspiration cytology
B. Excisional biopsy
C. Chest x-ray
D. Serologic tests

The correct answer is A. Fine-needle aspiration cytology


Explanation:
This patient has isolated cervical lymphadenopathy, which can be caused by
various benign or malignant conditions. The best initial investigation for this
patient is fine-needle aspiration cytology (FNAC), which is a minimally invasive
procedure that can obtain cellular samples from the lymph node and provide a
rapid diagnosis. FNAC can differentiate between reactive, inflammatory, infectious,
or neoplastic causes of lymphadenopathy, and guide further management.
Excisional biopsy is a more invasive procedure that can provide more tissue for
histologic and immunohistochemical analysis, but it is usually reserved for cases
where FNAC is inconclusive or unavailable. Chest x-ray and serologic tests are not
specific for the cause of lymphadenopathy, and would not be the best initial tests
for this patient.

46. A 30-year-old man presents to the clinic with fatigue, pallor, and fever. He has
a history of recent travel to sub-Saharan Africa, where he did not take any
prophylaxis. On physical examination, he has tachycardia, hypotension, and
splenomegaly. A blood smear shows ring forms and schizonts of Plasmodium
falciparum, with a parasitemia of 2%. He also has signs of cerebral malaria, such
as confusion, seizures, and coma. Which of the following is the most appropriate
treatment for this patient?
A. Coartem
B. Chloroquine
C. Artesunate
D. Mefloquine

The correct answer is C. Artesunate


Explanation:
This patient has severe malaria caused by P. falciparum, which is a life-threatening
infection that requires immediate treatment with intravenous or intramuscular
artesunate. Artesunate is a fast-acting artemisinin derivative that rapidly reduces
the parasite load and improves the clinical outcome. Coartem is a combination of
artemether and lumefantrine, which is an oral therapy for uncomplicated malaria.
Chloroquine is an antimalarial agent that is effective against P. vivax, P. ovale, and
P. malariae, but not against P. falciparum, which is resistant to it. Mefloquine is
another oral antimalarial agent that can be used for prophylaxis or treatment of
chloroquine-resistant malaria, but it is not recommended for severe malaria.
Primaquine is an antimalarial agent that can prevent relapses of P. vivax and P.
ovale by killing the dormant liver stages, but it has no effect on P. falciparum.

47. A 45-year-old man with HIV infection presents to the emergency department
with headache, fever, and neck stiffness. He has a history of poor adherence to
antiretroviral therapy, and his last CD4+ cell count was 50/mm3. A lumbar
puncture is performed, and the cerebrospinal fluid (CSF. analysis shows increased
opening pressure, increased protein, decreased glucose, and increased lymphocytes.
India ink stain of the CSF shows encapsulated yeast. Which of the following is the
most likely diagnosis?
A. Bacterial meningitis
B. Tuberculous meningitis
C. Cryptococcal meningitis
D. Viral meningitis

The correct answer is C. Cryptococcal meningitis


Explanation:
This patient has signs and symptoms of meningitis, which is an inflammation of
the meninges that can be caused by various infectious agents. The CSF analysis
and India ink stain confirm the diagnosis of cryptococcal meningitis, which is a
fungal infection caused by Cryptococcus neoformans, an encapsulated yeast that is
commonly seen in immunocompromised patients, especially those with HIV
infection. Cryptococcal meningitis is treated with amphotericin B and flucytosine,
followed by fluconazole. Tuberculous meningitis is another possible cause of
meningitis in HIV-infected patients, but it would show acid-fast bacilli on Ziehl-
Neelsen stain of the CSF.

48. A 60-year-old woman presents to the clinic with a sore throat, fever, and
malaise. She has a history of rheumatic heart disease and takes warfarin for
anticoagulation. On physical examination, she has pharyngeal erythema and
exudates, cervical lymphadenopathy, and a new systolic murmur. A throat swab is
obtained and sent for Gram stain and culture. The Gram stain shows lancet-shaped,
gram-positive diplococci. Which of the following is the most likely organism
causing this infection?
A. Streptococcus pyogenes
B. Staphylococcus aureus
C. Streptococcus pneumoniae
D. Neisseria gonorrhoeae

The correct answer is C. Streptococcus pneumoniae


Explanation:
This patient has signs and symptoms of pharyngitis, which is an inflammation of
the pharynx that can be caused by various bacterial or viral agents. The Gram stain
of the throat swab shows lancet-shaped, gram-positive diplococci, which are
characteristic of S. pneumoniae, a common cause of community-acquired
pneumonia, meningitis, and otitis media. S. pneumoniae can also cause pharyngitis,
especially in patients with underlying cardiac conditions, such as rheumatic heart
disease, that predispose them to endocarditis.
S. pyogenes is another common cause of pharyngitis, but it appears as gram-
positive cocci in chains on Gram stain. S. aureus is a gram-positive coccus in
clusters that can cause skin and soft tissue infections, as well as toxic shock
syndrome.
N. gonorrhoeae is a gram-negative diplococcus that can cause sexually transmitted
infections, such as urethritis, cervicitis, and pelvic inflammatory disease.

49. A 25-year-old man presents to the emergency department with headache, fever,
and altered mental status. He has a history of intravenous drug use and endocarditis.
On physical examination, he has focal neurological deficits and signs of increased
intracranial pressure. A head CT scan shows a ring-enhancing lesion in the right
frontal lobe. A lumbar puncture is contraindicated due to the risk of herniation.
Which of the following is the most appropriate empirical antibiotic regimen for
this patient?
A. Ceftriaxone, vancomycin, and metronidazole
B. Ceftriaxone and vancomycin
C. Cefepime and vancomycin
D. Meropenem and vancomycin

The correct answer is A. Ceftriaxone, vancomycin, and metronidazole


Explanation:
This patient has signs and symptoms of a brain abscess, which is a focal collection
of pus in the brain parenchyma that can be caused by various infectious agents.
The most common sources of brain abscess are infection of contiguous structures,
such as the sinuses, ears, or teeth, or hematogenous spread from distant sites, such
as the lungs, heart, or skin. In this patient, the most likely source is endocarditis
due to intravenous drug use, which can seed the brain with bacteria, such as
Staphylococcus aureus, Streptococcus species, or anaerobes. The most appropriate
empirical antibiotic regimen for a brain abscess is a combination of a third-
generation cephalosporin, such as ceftriaxone, to cover gram-positive and gram-
negative organisms, vancomycin to cover methicillin-resistant S. aureus, and
metronidazole to cover anaerobes.

50. A 2-year-old boy presents to the clinic with diarrhea, vomiting, and
dehydration. He has a history of rotavirus infection, which he contracted from his
daycare. On physical examination, he has sunken eyes, dry mucous membranes,
and poor skin turgor. His weight is 10 kg and his pulse is 120/min. A stool sample
is positive for rotavirus antigen. Which of the following is the most appropriate
management for this patient?
A. 75 mL/kg of oral rehydration solution over 4 hours
B. 100 mL/kg of oral rehydration solution over 4 hours
C. 150 mL/kg of oral rehydration solution over 4 hours
D. Antibiotics and antidiarrheals

The correct answer is A. 75 mL/kg of oral rehydration solution over 4 hours


Explanation:
This patient has signs and symptoms of acute gastroenteritis, which is an
inflammation of the gastrointestinal tract that can be caused by various infectious
agents, such as rotavirus. Acute gastroenteritis can cause dehydration, which is a
loss of water and electrolytes from the body. The best management for dehydration
is oral rehydration therapy, which is the administration of a solution containing
water, glucose, and salts to replace the fluid and electrolyte losses. The
recommended amount of oral rehydration solution for mild to moderate
dehydration is 75 mL/kg over 4 hours. The other options are either excessive,
insufficient, or inappropriate, and would not provide adequate rehydration for this
patient.

51. A 65-year-old man with a history of diabetes mellitus and chronic kidney
disease is admitted to the intensive care unit with pneumonia. He has a fever of
39.5°C, a heart rate of 130 beats/min, a blood pressure of 80/50 mm Hg, and a
respiratory rate of 28 breaths/min. He is receiving oxygen via a non-rebreather
mask and intravenous fluids and antibiotics. His laboratory tests show a white
blood cell count of 18,000/mm3, a blood glucose level of 250 mg/dL, a blood urea
nitrogen level of 60 mg/dL, and a creatinine level of 3.0 mg/dL. A blood culture is
positive for Staphylococcus aureus. Which of the following criteria is used to
diagnose septic shock in this patient?
A. Uncompensated septic shock
B. Compensated septic shock
C. Persistent hypotension despite adequate fluid resuscitation
D. Decreased mixed venous oxygen saturation

Answer is C. Persistent hypotension despite adequate fluid resuscitation


Explanation:
Septic shock is defined as a subset of sepsis in which underlying circulatory and
cellular/metabolic abnormalities are profound enough to substantially increase
mortality. The clinical criteria for septic shock are sepsis with hypotension that
does not respond to adequate fluid resuscitation (at least 30 mL/kg of crystalloid
fluids) and requiring vasopressors to maintain a mean arterial pressure of 65 mm
Hg or higher. Uncompensated and compensated septic shock are not official terms,
but they may refer to the degree of hemodynamic instability and organ dysfunction
in septic shock.

52. A 23-year-old woman is brought to the emergency department after a motor


vehicle collision. She was the driver and was wearing a seat belt. She has a cervical
spine fracture at C5 and is unable to move her arms or legs. She is conscious and
alert, but complains of difficulty breathing. Her vital signs are temperature 36.5°C,
heart rate 50 beats/min, blood pressure 70/40 mm Hg, and respiratory rate 22
breaths/min. On physical examination, she has a flaccid paralysis of all four limbs
and absent deep tendon reflexes. Her chest wall movement is decreased and her
breath sounds are diminished bilaterally. Her pupils are equal and reactive to light.
Which of the following is the most likely type of shock in this patient?
A. Neurogenic shock
B. Hypovolemic shock
C. Cardiogenic shock
D. Obstructive shock

The correct answer is A. Neurogenic shock


Explanation:
Neurogenic shock is a type of distributive shock that results from the loss of
sympathetic tone due to spinal cord injury, spinal anesthesia, or autonomic
neuropathy. The loss of sympathetic tone causes unopposed parasympathetic
stimulation, which leads to vasodilation, venous pooling, decreased venous return,
decreased cardiac output, and hypotension. Neurogenic shock also causes
bradycardia due to the lack of sympathetic stimulation of the heart and increased
vagal tone. Neurogenic shock can impair respiratory function due to the paralysis
of the intercostal and abdominal muscles, which are innervated by the thoracic and
lumbar spinal nerves. This patient has a cervical spine fracture that has disrupted
the spinal cord at the level of C5, which affects the phrenic nerve (C3-C5) that
innervates the diaphragm, as well as the nerves that innervate the upper and lower
extremities.
Cardiogenic shock is caused by a decrease in cardiac contractility due to
myocardial infarction, cardiomyopathy, arrhythmia, or valvular dysfunction. It is
characterized by tachycardia, hypotension, pulmonary edema, jugular venous
distension, and cool and cyanotic extremities. There is no evidence of cardiac
dysfunction in this patient.

53. A 75-year-old woman presents to the emergency department with hypotension,


tachycardia, and confusion. She has a history of osteoporosis and fell at home,
sustaining a left hip fracture. She was taken to the operating room for hip
replacement surgery, but developed severe bleeding intraoperatively and required
multiple blood transfusions. Her vital signs are temperature 36.5°C, heart rate 120
beats/min, blood pressure 70/40 mm Hg, and respiratory rate 24 breaths/min. On
physical examination, she has pale and cold skin, delayed capillary refill, and
decreased urine output. Her laboratory tests show a hemoglobin level of 7 g/dL, a
platelet count of 150,000/mm3, a prothrombin time of 15 seconds, and an activated
partial thromboplastin time of 35 seconds. Which of the following is the most
likely type of shock in this patient?
A. Hypovolemic shock
B. Neurogenic shock
C. Cardiogenic shock
D. Obstructive shock

The correct answer is A. Hypovolemic shock


Explanation:
Hypovolemic shock is a type of shock that results from a decrease in intravascular
volume due to hemorrhage, dehydration, or fluid loss. It is characterized by
tachycardia, hypotension, cold and clammy skin, oliguria, and altered mental status.
This patient has hypovolemic shock due to massive blood loss from her hip
fracture and surgery. The treatment for hypovolemic shock is fluid resuscitation
with crystalloid or colloid solutions, and blood transfusion if indicated. The other
types of shock are less likely in this patient, as they have different causes and
clinical features.

54. A 25-year-old man presents to the emergency department with dyspnea, chest
pain, and hypoxia. He has a history of left hip fracture and was placed in a cast two
days ago. On physical examination, he has tachypnea, tachycardia, and cyanosis.
His oxygen saturation is 85% on room air. A chest x-ray shows bilateral diffuse
infiltrates. An arterial blood gas shows a pH of 7.35, a PaCO2 of 35 mm Hg, a
PaO2 of 60 mm Hg, and a bicarbonate of 22 mEq/L. Which of the following is the
most likely diagnosis?
A. Fat embolism syndrome
B. Pulmonary embolism
C. Acute respiratory distress syndrome
D. Congestive heart failure

The correct answer is A. Fat embolism syndrome


Explanation:
Fat embolism syndrome is a rare complication of long bone fractures, orthopedic
surgery, or liposuction, that occurs when fat globules enter the circulation and
cause mechanical obstruction and inflammatory reaction in the pulmonary and
systemic capillaries. It is characterized by a triad of respiratory distress, neurologic
dysfunction, and petechial rash, which usually manifest within 24 to 72 hours after
the injury or procedure. The diagnosis is based on clinical suspicion and supportive
findings, such as hypoxemia, diffuse infiltrates on chest x-ray, fat droplets in urine
or sputum, and thrombocytopenia. The treatment is supportive, with oxygen,
mechanical ventilation, and corticosteroids.
Pulmonary embolism is another possible complication of long bone fractures, but it
would show a wedge-shaped infarct or pleural effusion on chest x-ray, and a
positive D-dimer test.
Acute respiratory distress syndrome is a non-cardiogenic pulmonary edema that
can be caused by various insults, such as sepsis, trauma, or aspiration, but it is
unlikely in this patient without a clear trigger.

55. A 35-year-old woman presents to the clinic with a rash, joint pain, and fever.
She has a history of syphilis and was treated with intramuscular penicillin G two
days ago. On physical examination, she has a diffuse maculopapular rash, tender
swollen joints, and enlarged lymph nodes. Her vital signs are temperature 38.5°C,
heart rate 100 beats/min, blood pressure 110/70 mm Hg, and respiratory rate 18
breaths/min. A rapid plasma reagin test is positive. Which of the following is the
most likely explanation for this patient's condition?
A. Anaphylactic shock
B. Septic shock
C. Jarisch-Herxheimer reaction
D. Serum sickness

The correct answer is C. Jarisch-Herxheimer reaction


Explanation:
Jarisch-Herxheimer reaction is an acute febrile reaction that occurs within the first
24 hours of antibiotic treatment for syphilis or other spirochetal infections, such as
Lyme disease or leptospirosis. It is caused by the release of endotoxins from the
dying bacteria, which trigger an inflammatory response. It is characterized by fever,
chills, headache, myalgia, arthralgia, rash, and lymphadenopathy. The diagnosis is
based on clinical presentation and history of antibiotic therapy. The treatment is
supportive, with antipyretics, analgesics, and fluids.
Anaphylactic shock is a type of distributive shock that results from a severe
allergic reaction mediated by IgE antibodies. It is characterized by urticaria,
angioedema, bronchospasm, laryngeal edema, and hypotension.
Septic shock is a type of distributive shock that results from a systemic
inflammatory response to an infection. It is characterized by fever, tachycardia,
hypotension, organ dysfunction, and positive blood cultures.
Serum sickness is a type of hypersensitivity reaction that occurs after exposure to
foreign proteins, such as animal antiserum or certain drugs. It is characterized by
fever, rash, arthralgia, lymphadenopathy, and nephritis, and it usually manifests 7
to 14 days after exposure.

56. Which of the following is the most likely diagnosis for a patient who presents
with the following stool sample?
A. Hookworm infection
B. Schistosomiasis
C. Strongyloidiasis
D. Giardiasis

The correct answer is A. Hookworm infection


Explanation:
Hookworm infection is caused by the nematodes Necator americanus and
Ancylostoma duodenale, which are endemic in tropical and subtropical regions.
The infection is acquired by skin penetration of the larvae from contaminated soil.
The larvae migrate through the bloodstream to the lungs, where they are coughed
up and swallowed. The adult worms attach to the small intestine and feed on blood,
causing iron deficiency anemia, malnutrition, and abdominal pain. The diagnosis is
made by finding eggs or larvae in the stool. The treatment is albendazole or
mebendazole.

57. A 55-year-old man with a history of hypertension and diabetes mellitus is


admitted to the hospital with acute kidney injury. He has oliguria, edema, and
uremic symptoms. His laboratory tests show a blood urea nitrogen level of 80
mg/dL, a creatinine level of 4.0 mg/dL, a pH of 7.15, a bicarbonate level of 12
mEq/L, and a potassium level of 6.5 mEq/L. Which of the following is the most
immediate cause of death in patients with acute kidney injury?
A. Encephalopathy
B. Metabolic acidosis
C. Hyperkalemia
D. Pulmonary edema

The correct answer is C. Hyperkalemia


Explanation:
Hyperkalemia is a high level of potassium in the blood, which can cause cardiac
arrhythmias and arrest. It is a common and life-threatening complication of acute
kidney injury, which is a sudden loss of kidney function due to various causes. The
treatment is calcium gluconate, insulin, glucose, sodium bicarbonate, and dialysis.

58. A 10-year-old boy presents to the clinic with a skin infection on his right leg.
He has a history of a minor cut that he sustained while playing soccer. On physical
examination, he has a 5-cm erythematous, warm, and tender area on his lower leg,
with a central area of pus and necrosis. A culture of the wound grows group A
streptococcus. He is treated with oral penicillin and the infection resolves. Two
weeks later, he develops hematuria, proteinuria, and hypertension. A renal biopsy
shows glomerular inflammation and deposition of immune complexes. Which of
the following complications of group A streptococcal infection is not preventable
by antibiotic therapy?
A. Acute glomerulonephritis
B. Acute rheumatic fever
C. Scarlet fever
D. Post-streptococcal glomerulonephritis

The correct answer is A. Acute glomerulonephritis


Explanation:
This is an immune-mediated kidney injury that occurs 1-4 weeks after a strep
infection. It is not prevented by antibiotics because it is caused by preformed
antibodies that cross-react with the kidney. The other options are all preventable by
antibiotics because they are caused by the bacteria or its toxins.

59. A 59-year-old man with a history of diabetes mellitus and retinopathy presents
with proteinuria and elevated serum creatinine. What is the best indicator of the
severity of his nephropathy?
A. Urine albumin-to-creatinine ratio
B. Glomerular filtration rate
C. Blood urea nitrogen
D. Serum potassium

Answer: B. Glomerular filtration rate


Explanation:
Glomerular filtration rate (GFR) is the best indicator of the severity of nephropathy
in diabetic patients, as it reflects the degree of renal function impairment. Urine
albumin-to-creatinine ratio is useful for screening and diagnosis of nephropathy,
but not for staging. Blood urea nitrogen, serum potassium, and urine osmolality are
affected by other factors besides renal function, such as hydration status, diet, and
medications.

60. A 60-year-old woman with a history of recurrent urinary tract infections


presents with sudden onset of severe left flank pain that radiates to the groin. She
also has nausea, vomiting, and hematuria. What is the best investigation for her
condition?
A. CT scan
B. Ultrasound
C. Intravenous pyelography
D. Urine culture

Answer: A. CT scan
Explanation:
CT scan is the best investigation for ureteric colic, as it can detect the presence,
location, and size of urinary stones, as well as any complications such as
hydronephrosis or infection. Ultrasound is less sensitive and specific than CT scan,
and may miss small or radiolucent stones. Intravenous pyelography is an invasive
procedure that involves exposure to contrast and radiation, and may be
contraindicated in patients with renal impairment or allergy. Urine culture is
indicated to rule out infection, but does not confirm the diagnosis of ureteric colic.
Cystoscopy is an endoscopic procedure that can be used to remove stones from the
bladder or lower ureter, but is not a diagnostic test.

61. A 25-year-old man presents with urethral discharge and knee joint swelling. He
reports having unprotected sex with multiple partners in the past month. On
physical exam, there is purulent discharge at the urethral meatus and a warm,
tender, and swollen left knee. Synovial fluid analysis reveals numerous neutrophils
and intracellular gram-negative diplococci. What is the most likely diagnosis?
A. Gonococcal arthritis
B. Reactive arthritis
C. Septic arthritis
D. Gouty arthritis

Answer: A. Gonococcal arthritis


Explanation:
Gonococcal arthritis is a complication of disseminated gonococcal infection, which
typically affects young, sexually active individuals. It presents with a triad of
tenosynovitis, dermatitis, and polyarthralgia, or with a monoarticular septic
arthritis. The diagnosis is confirmed by culture or nucleic acid amplification test of
the urethral discharge and/or synovial fluid. Treatment involves intravenous or
intramuscular ceftriaxone and oral azithromycin.

62. A 40-year-old woman with a history of rheumatoid arthritis presents with


swelling of two fingers, including the thumb. She says that the swelling started a
few days ago and is associated with pain and difficulty in moving the fingers. On
physical exam, there is fusiform swelling of the flexor tendon sheaths of the index
and thumb of the right hand. There is no erythema, warmth, or purulence. What is
the most likely diagnosis?
A. Tenosynovitis
B. Cellulitis
C. Osteomyelitis
D. Dupuytren contracture
Answer: A. Tenosynovitis
Explanation:
Tenosynovitis is an inflammation of the synovial membrane that surrounds the
tendons. It can be caused by various conditions, such as rheumatoid arthritis, gout,
diabetes, or trauma. It presents with swelling, pain, and reduced range of motion of
the affected tendon. Treatment involves rest, immobilization, anti-inflammatory
drugs, and corticosteroid injections.

63. A 12-year-old boy falls from his bicycle and injures his left wrist. He
complains of severe pain and deformity of the wrist. On physical exam, there is
swelling and tenderness over the distal radius. An x-ray of the wrist shows a
fracture of the distal radial metaphysis with an upward displacement of the distal
fragment. What is the most likely type of fracture?
A. Salter-Harris type I
B. Salter-Harris type II
C. Salter-Harris type III
D. Salter-Harris type IV

Answer: B. Salter-Harris type II


Explanation:
Salter-Harris fractures are fractures involving the growth plate (physis) of long
bones in children. They are classified into five types based on the location and
extent of the fracture. Type I fractures are transverse fractures through the physis.
Type II fractures are fractures through the physis and the metaphysis. Type III
fractures are fractures through the physis and the epiphysis. Type IV fractures are
fractures through the metaphysis, physis, and epiphysis. Type V fractures are
compression fractures of the physis. The mnemonic SALTR can be used to
remember the types: S = Slip (type I), A = Above (type II), L = Lower (type III), T
= Through (type IV), R = Ram (type V).
64. A 65-year-old man with a history of hypertension and atrial fibrillation presents
with sudden onset of left-sided weakness and slurred speech. He was last seen
normal 3 hours ago. On physical exam, he has a blood pressure of 220/120 mmHg,
a pulse of 110 beats/min, and an irregularly irregular rhythm. He has a right gaze
preference, left facial droop, left hemiparesis, and left hemisensory loss. A non-
contrast head CT shows a large right basal ganglia hemorrhage with
intraventricular extension. What is the most appropriate antihypertensive agent to
use in this patient?
A. Hydralazine
B. Nifedipine
C. Nicardipine
D. Labetalol

Answer: C. Nicardipine
Explanation:
Nicardipine is a calcium channel blocker that can be used to lower blood pressure
in patients with acute intracerebral hemorrhage. It has a rapid onset and offset of
action, and can be titrated easily to achieve the target blood pressure. The
American Heart Association/American Stroke Association guidelines recommend
lowering the systolic blood pressure to < 140 mmHg in patients with intracerebral
hemorrhage who have a systolic blood pressure of 150-220 mmHg. Hydralazine
and nifedipine are not recommended because they can cause reflex tachycardia and
cerebral vasodilation, which may worsen the bleeding. Labetalol and enalapril are
alternative agents, but they have a slower onset and longer duration of action than
nicardipine.

65. A 50-year-old man presents with a history of involuntary shaking of both his
hands for the past 10 years. He says that the tremor is worse when he tries to write,
eat, or drink, and that it improves with alcohol consumption. He also has a mild
tremor of his head and voice. His father had a similar condition. On physical exam,
he has a bilateral postural tremor of his hands with a frequency of 8 Hz. There is
no rigidity, bradykinesia, or ataxia. What is the most effective pharmacological
treatment for his condition?
A. Propranolol
B. Levodopa
C. Clonazepam
D. Primidone

Answer: A. Propranolol
Explanation:
Propranolol is a non-selective beta-blocker that can reduce the amplitude and
frequency of essential tremor. It is the first-line pharmacological treatment for
essential tremor, especially when it affects the hands, head, or voice. Levodopa is
the mainstay of treatment for parkinsonian tremor, but has no effect on essential
tremor. Clonazepam and primidone are second-line agents for essential tremor, but
they have more side effects than propranolol, such as sedation, cognitive
impairment, and dependence. Botulinum toxin can be used to treat focal or
segmental dystonic tremor, but not essential tremor.

66. A 6-year-old boy presents with a generalized tonic-clonic seizure that lasts for
5 minutes. He has no history of seizures or neurological disorders. He has a fever
of 38.5°C and complains of a sore throat. His cerebrospinal fluid (CSF. analysis is
normal. What is the most likely diagnosis?
A. Tonsillopharyngitis
B. Viral meningitis
C. Viral encephalitis
D. Bacterial meningitis

Answer: A. Tonsillopharyngitis
Explanation:
Tonsillopharyngitis is a common cause of febrile seizures in children, especially
those between 6 months and 5 years of age. Febrile seizures are usually benign and
self-limited, and do not require antiepileptic drugs. CSF analysis is normal in
febrile seizures, unless there is an underlying CNS infection. Viral and bacterial
meningitis and encephalitis typically cause abnormal CSF findings, such as
elevated white blood cells, protein, and lactate, and decreased glucose.

67. A 65-year-old woman with a history of hypertension and diabetes mellitus


presents with sudden onset of left-sided weakness and dysarthria. She was last seen
normal 2 hours ago. On physical exam, she has a blood pressure of 200/110 mmHg,
a pulse of 90 beats/min, and a regular rhythm. She has a right gaze preference, left
facial droop, left hemiparesis, and left hemisensory loss. A non-contrast head CT
shows a small hypodense area in the right hemisphere. What is the most likely
location of the lesion?
A. Thalamus
B. Corona radiata
C. Basal ganglia
D. Internal capsule

Answer: D. Internal capsule


Explanation:
The internal capsule is a white matter structure that contains the corticospinal and
corticobulbar tracts, which mediate voluntary motor function. A lesion of the
internal capsule, usually due to lacunar infarction, causes pure motor hemiparesis
and hemisensory loss, without cortical signs such as aphasia, neglect, or visual
field defects. The corona radiata is a similar structure, but it is more extensive and
involves both the motor and sensory fibers. A lesion of the corona radiata would
cause similar symptoms, but it would also affect the cortical areas and produce a
larger infarct on CT scan. The basal ganglia, thalamus, and brainstem are
subcortical structures that have different functions and clinical manifestations.

68. A 55-year-old man with a history of smoking and hyperlipidemia presents with
a transient episode of right arm weakness and slurred speech that resolved
spontaneously after 15 minutes. He denies any headache, chest pain, or palpitations.
On physical exam, he has a blood pressure of 160/90 mmHg, a pulse of 80
beats/min, and a regular rhythm. He has no focal neurological deficits. A carotid
Doppler ultrasound shows a 70% stenosis of the left internal carotid artery. What is
the most important modifiable risk factor for his condition?
A. Hypertension
B. Dyslipidemia
C. Smoking
D. Diabetes mellitus

Answer: A. Hypertension
Explanation:
Hypertension is the most important modifiable risk factor for stroke, as it
contributes to atherosclerosis, endothelial damage, and vascular remodeling. The
patient has a history of a transient ischemic attack (TIA), which is a brief episode
of focal neurological dysfunction due to ischemia, without infarction. TIAs are
often caused by emboli from atherosclerotic plaques in the carotid arteries. Other
modifiable risk factors for stroke include dyslipidemia, smoking, diabetes mellitus,
obesity, atrial fibrillation, and oral contraceptive use.
69. A 45-year-old woman presents with progressive weakness and numbness of all
four limbs for the past two weeks. She also has difficulty breathing and swallowing.
She has a history of diarrhea and fever one month ago, which resolved
spontaneously. On physical exam, she has quadriparesis with areflexia and
respiratory insufficiency. Her sensory exam is normal. A lumbar puncture shows
elevated protein and normal cell count in the cerebrospinal fluid. What is the most
likely diagnosis?
A. Guillain-Barré syndrome
B. Multiple sclerosis
C. Myasthenia gravis
D. Poliomyelitis

Answer: A. Guillain-Barré syndrome


Explanation:
Guillain-Barré syndrome (GBS) is an acute inflammatory demyelinating
polyneuropathy that causes ascending symmetric weakness, areflexia, and
autonomic dysfunction. It is often preceded by an infection, such as Campylobacter
jejuni, or a vaccination. It can affect the cranial nerves, causing facial weakness,
dysphagia, and ophthalmoplegia. It can also cause respiratory failure due to
diaphragmatic paralysis. The cerebrospinal fluid typically shows
albuminocytologic dissociation, which is elevated protein and normal cell count.
The diagnosis is confirmed by nerve conduction studies and electromyography.
Treatment involves intravenous immunoglobulin or plasmapheresis.

70. A 35-year-old man presents with a stab wound to the right side of his neck. He
has severe pain and bleeding from the wound. On physical exam, he has right-
sided weakness and loss of proprioception and vibration sensation in his right arm
and leg. He also has loss of pain and temperature sensation in his left arm and leg.
His cranial nerve exam is normal. What is the most likely diagnosis?
A. Brown-Séquard syndrome
B. Cauda equina syndrome
C. Central cord syndrome
D. Horner syndrome

Answer: A. Brown-Séquard syndrome


Explanation:
Brown-Séquard syndrome is a hemisection of the spinal cord that causes ipsilateral
motor paralysis and loss of proprioception and vibration sensation, and
contralateral loss of pain and temperature sensation below the level of the lesion. It
is usually caused by trauma, such as a penetrating injury, or spinal cord
compression. The cranial nerves are spared, unless the lesion is at the level of the
medulla. Horner syndrome (ptosis, miosis, and anhidrosis) may also occur if the
lesion affects the cervical sympathetic chain. Wallenberg syndrome is a lateral
medullary infarction that causes ipsilateral loss of pain and temperature sensation
in the face, and contralateral loss of pain and temperature sensation in the body,
along with other cranial nerve and cerebellar signs.

71. A 45-year-old man comes to the emergency department because of a sudden


onset of severe vertigo, nausea, and vomiting. He also has difficulty walking and
keeping his balance. He has no history of ear infections, head trauma, or
neurological disorders. Which of the following is the most appropriate initial test to
perform in this patient?
A. Nystagmus
B. Coordination
C. Audiometry
D. Caloric stimulation
E. Dix-Hallpike maneuver
Answer: A. Nystagmus
Explanation:
Nystagmus is an involuntary rhythmic movement of the eyes that can be horizontal,
vertical, or rotary. It can be a sign of vestibular dysfunction, which can cause
vertigo, nausea, and vomiting. Nystagmus can be elicited by observing the patient's
eye movements in different directions of gaze, or by performing the head impulse
test, which involves rapidly rotating the patient's head to one side while observing
the eyes for corrective saccades. Nystagmus can help differentiate between
peripheral and central causes of vertigo. Peripheral causes, such as benign
paroxysmal positional vertigo (BPPV., vestibular neuritis, or Meniere disease,
typically produce a horizontal or rotary nystagmus that is inhibited by visual
fixation. Central causes, such as cerebellar stroke, brainstem tumor, or multiple
sclerosis, typically produce a vertical or rotary nystagmus that is not inhibited by
visual fixation.

72. A 35-year-old woman with a history of HIV infection presents with headache,
confusion, and seizures. She has been noncompliant with her antiretroviral therapy
for the past year. A brain MRI shows a single ring-enhancing lesion in the right
frontal lobe. Which of the following is the most likely diagnosis?
A. Primary CNS lymphoma
B. Toxoplasmosis
C. Tuberculoma
D. Neurocysticercosis
E. Glioblastoma multiforme

Answer: A. Primary CNS lymphoma


Explanation:
Primary CNS lymphoma is a rare type of non-Hodgkin lymphoma that originates
in the brain, spinal cord, or meninges. It is more common in immunocompromised
patients, such as those with HIV/AIDS, organ transplant recipients, or patients
receiving immunosuppressive drugs. Primary CNS lymphoma typically presents
with headache, confusion, seizures, or focal neurological deficits. Brain MRI
usually shows a single or multiple ring-enhancing lesions, often with
periventricular involvement. The diagnosis can be confirmed by brain biopsy,
which shows malignant lymphoid cells that are positive for CD20 and EBV. The
treatment is a combination of high-dose methotrexate, rituximab, and whole-brain
radiation therapy.
73. A 50-year-old man with a history of hypertension and diabetes mellitus
presents with blurred vision, headache, and nausea. He has been taking
hydrochlorothiazide and metformin for the past 5 years. On physical examination,
his blood pressure is 180/110 mmHg, pulse is 90 beats/min, and respirations are 16
breaths/min. His pupils are equal and reactive to light. Funduscopic examination
shows papilledema. A slit-lamp examination would most likely reveal which of the
following findings in this patient?
A. Anterior uveitis
B. Cataract
C. Glaucoma
D. Retinal detachment
E. Retinal hemorrhage

Answer: E. Retinal hemorrhage


Explanation:
Retinal hemorrhage is a condition in which blood leaks from the retinal vessels
into the retina or the vitreous humor. It can be caused by various conditions, such
as hypertension, diabetes mellitus, trauma, coagulopathy, or retinal vein occlusion.
Retinal hemorrhage can cause blurred vision, floaters, or scotomas. A slit-lamp
examination is a technique that uses a microscope and a high-intensity light source
to examine the anterior and posterior segments of the eye. A slit-lamp examination
can detect retinal hemorrhage as red patches or spots on the retina. Other findings
that can be seen with a slit-lamp examination include anterior uveitis
(inflammation of the iris and ciliary body., cataract (clouding of the lens.,
glaucoma (increased intraocular pressure., and retinal detachment (separation of
the retina from the choroid).

74. A 40-year-old woman comes to the clinic because of a painless lump in her left
breast that she noticed 2 months ago. She has no family history of breast cancer.
On physical examination, the lump is firm, mobile, and well-circumscribed. It
measures about 3 cm in diameter and is located in the upper outer quadrant of the
left breast. There is no nipple discharge, skin changes, or axillary
lymphadenopathy. A mammogram shows a round mass with smooth margins and
no calcifications. A biopsy of the mass is most likely to show which of the
following histological features?
A. Ductal carcinoma in situ
B. Invasive ductal carcinoma
C. Invasive lobular carcinoma
D. Fibroadenoma

Answer: D. Fibroadenoma
Explanation:
Fibroadenoma is a benign tumor of the breast that arises from the stromal and
epithelial components of the terminal duct lobular unit. It is the most common
benign breast tumor in young women. Fibroadenoma typically presents as a
painless, mobile, well-circumscribed mass that may vary in size with the menstrual
cycle or pregnancy. Mammography usually shows a round or oval mass with
smooth margins and no calcifications. Histologically, fibroadenoma is composed
of a proliferation of glandular and fibrous tissue, forming a biphasic pattern of
ducts and stroma. The stroma is usually cellular and may show myxoid or hyaline
degeneration. The ducts are lined by a single layer of epithelial cells and may show
apocrine metaplasia.

75. A 60-year-old man comes to the clinic because of abdominal pain, weight loss,
and jaundice. He has a history of chronic pancreatitis and heavy alcohol use. On
physical examination, he has a palpable mass in the epigastrium and a palpable
gallbladder. Laboratory tests show elevated serum levels of amylase, lipase,
bilirubin, and alkaline phosphatase. A CT scan of the abdomen shows a mass in the
head of the pancreas with dilatation of the common bile duct and the pancreatic
duct. A biopsy of the mass is most likely to show which of the following molecular
features?
A. BRCA1 mutation
B. BRCA2 mutation
C. KRAS mutation
D. TP53 mutation

Answer: C. KRAS mutation


Explanation:
Pancreatic cancer is a malignant neoplasm that arises from the exocrine or
endocrine cells of the pancreas. It is the fourth leading cause of cancer-related
death in the United States. The most common type of pancreatic cancer is
pancreatic ductal adenocarcinoma, which accounts for about 85% of cases.
Pancreatic ductal adenocarcinoma typically presents with abdominal pain, weight
loss, and jaundice due to obstruction of the bile duct by the tumor. KRAS is a
member of the RAS family of GTPases, which regulate cell proliferation,
differentiation, and survival. KRAS mutation leads to constitutive activation of the
downstream signaling pathways, such as MAPK and PI3K/AKT, resulting in
increased cell growth and resistance to apoptosis.
BRCA1 and BRCA2 mutations are more commonly associated with breast and
ovarian cancers, but they can also increase the risk of pancreatic cancer.
VHL mutation is associated with von Hippel-Lindau syndrome, which predisposes
to renal cell carcinoma, hemangioblastoma, and pheochromocytoma.

76. A 25-year-old woman comes to the clinic because of a painful lump in her right
breast that she noticed 3 days ago. She is breastfeeding her 2-month-old infant. On
physical examination, the lump is tender, warm, and erythematous. There is no
nipple discharge or axillary lymphadenopathy. Which of the following is the most
appropriate initial management for this patient?
A. Incision and drainage
B. Start antibiotics
C. Stop breastfeeding
D. Fine-needle aspiration
Answer: B. Start antibiotics
Explanation:
Breast abscess is a localized collection of pus in the breast tissue, usually caused
by bacterial infection. It is more common in lactating women, due to nipple trauma,
milk stasis, or mastitis. Breast abscess typically presents with a painful, warm, and
erythematous lump in the breast, often associated with fever and malaise. The
initial management of breast abscess is to start antibiotics, such as dicloxacillin or
clindamycin, and to continue breastfeeding or pumping to prevent milk stasis and
engorgement. If the abscess does not resolve with antibiotics, then incision and
drainage may be indicated. Fine-needle aspiration can be used to confirm the
diagnosis of abscess and to obtain a culture of the pus. Mammography is not
indicated in lactating women, as it can cause false-positive results due to increased
breast density.

77. A 45-year-old woman comes to the clinic for a routine gynecological


examination. She has no complaints or symptoms. She has a history of human
papillomavirus (HPV) infection and abnormal Pap smears. She has never been
pregnant. On physical examination, the cervix appears normal. A colposcopy is
performed and shows an area of acetowhite epithelium involving the upper two-
thirds of the vagina. A biopsy of the lesion is most likely to show which of the
following histological features?
A. Stage 1 cervical cancer
B. Stage 2 cervical cancer
C. Vaginal intraepithelial neoplasia
D. Vaginal adenocarcinoma

Answer: C. Vaginal intraepithelial neoplasia


Explanation:
Vaginal intraepithelial neoplasia (VAIN) is a precancerous condition of the vagina,
characterized by dysplasia of the squamous epithelium. It is usually caused by
HPV infection, especially HPV types 16 and 18. VAIN is often asymptomatic and
detected incidentally during colposcopy or Pap smear. Colposcopy can reveal areas
of acetowhite epithelium, which are abnormal epithelial cells that turn white after
application of acetic acid. Biopsy can confirm the diagnosis of VAIN and
determine the grade of dysplasia. VAIN is classified into three grades: VAIN 1
(mild dysplasia., VAIN 2 (moderate dysplasia), and VAIN 3 (severe dysplasia or
carcinoma in situ). The treatment of VAIN depends on the grade, extent, and
location of the lesion, and may include topical therapy, laser ablation, or surgical
excision. VAIN is a risk factor for developing invasive vaginal cancer, which is a
rare malignancy that usually presents with abnormal vaginal bleeding, discharge,
or mass.
Vaginal cancer can be staged according to the TNM system, where stage 1 means,
the tumor is confined to the vagina, stage 2 means the tumor has invaded the
subvaginal tissue or the paravaginal tissue, stage 3 means the tumor has extended
to the pelvic wall or the lower third of the urethra or the vagina, and stage 4 means
the tumor has invaded the bladder, rectum, or beyond the true pelvis.

78. A 30-year-old woman comes to the clinic for a routine gynecological


examination. She has no complaints or symptoms. She is sexually active with one
partner and uses oral contraceptives. She has no history of sexually transmitted
infections or abnormal Pap smears. She has never been vaccinated against human
papillomavirus (HPV.. Which of the following is the most effective way to prevent
cervical cancer in this patient?
A. HPV vaccination
B. Pap smear
C. Condom use
D. Hysterectomy

Answer: A. HPV vaccination


Explanation:
HPV vaccination is the most effective way to prevent cervical cancer, as HPV
infection is the main cause of cervical cancer. HPV is a sexually transmitted virus
that can infect the cervix and cause dysplasia and neoplasia of the squamous
epithelium. HPV types 16 and 18 are responsible for about 70% of cervical cancers.
HPV vaccination can protect against these high-risk types, as well as other types
that cause genital warts and other cancers. HPV vaccination is recommended for
all girls and boys aged 11 to 12 years, with catch-up vaccination for those aged 13
to 26 years who have not been previously vaccinated. HPV vaccination can also be
given to adults aged 27 to 45 years who are at increased risk of HPV infection,
such as those who are sexually active with multiple partners, immunocompromised,
or have a history of abnormal Pap smears. HPV vaccination does not eliminate the
need for cervical cancer screening, as it does not protect against all HPV types. Pap
smear is a screening test that can detect cervical dysplasia and cancer by examining
the cells of the cervix.
Pap smear is recommended for all women aged 21 to 65 years, with different
intervals depending on the age and the results of previous tests. Pap smear can also
be combined with HPV testing for women aged 30 to 65 years, which can increase
the sensitivity and specificity of the screening.
Condom use can reduce the risk of HPV transmission, but it is not a reliable
method of prevention, as HPV can infect areas that are not covered by the condom.
Hysterectomy is the surgical removal of the uterus, which can be a treatment
option for cervical cancer, but it is not a preventive measure.

79. A 50-year-old woman comes to the clinic because of heavy and prolonged
menstrual bleeding for the past 6 months. She also has pelvic pain, urinary
frequency, and constipation. She has a history of fibroids and endometriosis. She
has two children and does not want to have more. On physical examination, the
uterus is enlarged and irregular. A pelvic ultrasound shows a 6-cm submucosal
fibroid in the posterior wall of the uterus. Which of the following is the most
appropriate management for this patient?
A. Hysterectomy
B. Cone biopsy
C. Myomectomy
D. Uterine artery embolization

Answer: A. Hysterectomy
Explanation:
Hysterectomy is the surgical removal of the uterus, which can be a definitive
treatment for fibroids, especially in women who have completed childbearing and
have severe symptoms. Fibroids are benign tumors of the smooth muscle cells of
the uterus, which can cause abnormal uterine bleeding, pelvic pain, urinary
frequency, constipation, and infertility. Fibroids can be classified according to their
location in the uterus: submucosal (under the endometrium), intramural (within the
myometrium), or subserosal (under the peritoneum). Submucosal fibroids are more
likely to cause heavy and prolonged menstrual bleeding, as they distort the
endometrial cavity and interfere with the normal contraction of the uterus.Cone
biopsy treats cervical dysplasia or cancer, not fibroids.
Myomectomy removes fibroids while preserving the uterus, which can be an
option for women who wish to retain their fertility or avoid hysterectomy.
However, myomectomy has the risk of recurrence of fibroids, as well as the risk of
bleeding, infection, and adhesions.
Uterine artery embolization, a minimally invasive procedure, blocks blood supply
to fibroids, causing them to shrink. It carries risks including infection and impaired
fertility.

80. A 25-year-old woman comes to the clinic because of recurrent intrusive


thoughts and compulsive behaviors that interfere with her daily functioning. She
spends hours every day checking and rechecking her locks, appliances, and emails.
She also has to arrange her books and clothes in a specific order and symmetry.
She feels anxious and distressed if she does not perform these rituals. She has no
history of trauma or substance abuse. She is single and works as a librarian. Which
of the following is the most likely risk factor for her obsessive-compulsive disorder
(OCD.?
A. Single
B. Socioeconomic status
C. Gender
D. Personality

Answer: D. Personality
Explanation:
OCD is a mental disorder characterized by persistent and unwanted thoughts
(obsessions) and repetitive behaviors (compulsions) that cause significant distress
and impairment. The exact cause of OCD is unknown, but several risk factors have
been identified, such as genetics, environmental factors, and brain alterations.
Personality is one of the risk factors that may influence the development of OCD.
People who score high on measures of neuroticism, perfectionism, or harm
avoidance may be more prone to OCD. Neuroticism is a personality trait that
reflects the tendency to experience negative emotions, such as anxiety, anger, or
depression. Perfectionism is a personality trait that reflects the tendency to set and
pursue excessively high standards, and to be overly critical of oneself and others.
Harm avoidance is a personality trait that reflects the tendency to avoid situations
that may cause physical or psychological discomfort, and to be cautious and fearful
of uncertainty. These personality traits may predispose people to OCD by making
them more sensitive to stress, more vulnerable to intrusive thoughts, and more
likely to engage in compulsive behaviors to reduce anxiety. Single is not a risk
factor for OCD, as there is no evidence that marital status affects the prevalence or
severity of OCD. Socioeconomic status is also not a risk factor for OCD, as there is
no consistent association between income, education, or occupation and OCD.
Gender is a risk factor for OCD only in childhood, as males are more likely to
develop early-onset OCD, but not in adulthood, as the gender ratio is equal. Family
history is a risk factor for OCD, as having parents or other relatives with OCD
increases the likelihood of developing OCD, especially if the onset is in childhood
or adolescence. However, in this case, the patient has no family history of OCD, so
this is not the most likely risk factor.

81. A 35-year-old man comes to the clinic because of low mood, low self-esteem,
and poor appetite for the past 4 years. He also has difficulty sleeping, concentrating,
and making decisions. He has never had any psychotic symptoms or suicidal
thoughts. He has no history of manic or hypomanic episodes. He has tried several
antidepressants without significant improvement. Which of the following is the
most likely diagnosis?
A. Persistent depressive disorder
B. Major depressive disorder with psychotic features
C. Schizoaffective disorder, depressive type
D. Schizophrenia
Answer: A. Persistent depressive disorder
Explanation:
Persistent depressive disorder (PDD), also known as dysthymia, is a mental
disorder characterized by a chronic depressed mood, lasting for at least 2 years,
and accompanied by at least two other symptoms, such as low energy, poor
concentration, insomnia or hypersomnia, low self-esteem, or hopelessness. PDD
does not have psychotic features, and is less severe but more chronic than major
depressive disorder (MDD).
MDD is a mental disorder characterized by at least one episode of depressed mood
or loss of interest or pleasure in most activities, lasting for at least 2 weeks, and
accompanied by at least four other symptoms, such as low energy, poor
concentration, insomnia or hypersomnia, weight loss or gain, psychomotor
agitation or retardation, feelings of worthlessness or guilt, suicidal ideation, or
psychotic features. MDD with psychotic features is more severe and has a poorer
prognosis than MDD without psychotic features.
Schizoaffective disorder is a mental disorder characterized by a combination of
psychotic symptoms, such as delusions or hallucinations, and mood symptoms,
such as depression or mania. The psychotic symptoms must be present for at least
2 weeks in the absence of mood symptoms, and the mood symptoms must be
present for a substantial portion of the illness.

82. A 28-year-old woman with a history of bipolar disorder comes to the clinic
because she is 8 weeks pregnant. She has been taking valproic acid for the past 2
years, but she stopped it as soon as she found out she was pregnant. She is worried
about the risk of birth defects and the recurrence of her mood symptoms. She asks
the physician if there is any safe antipsychotic medication that she can take during
pregnancy. Which of the following is the most appropriate response?
A. Lithium
B. Olanzapine
C. Quetiapine
D. Risperidone
Answer: C. Quetiapine
Explanation:
Quetiapine is an atypical antipsychotic medication that can be used to treat bipolar
disorder, schizophrenia, or major depressive disorder. Quetiapine has a low risk of
causing birth defects or adverse effects on the fetus or the newborn, and is
considered relatively safe during pregnancy. Quetiapine can also be used during
breastfeeding, as it has a low concentration in breast milk and a low oral
bioavailability in infants. Quetiapine can cause sedation, weight gain, metabolic
syndrome, or orthostatic hypotension, but these side effects are usually mild and
manageable.
Lithium is a mood stabilizer that can be used to treat bipolar disorder, especially
manic episodes. Lithium has a high risk of causing birth defects, such as Ebstein
anomaly (a congenital heart defect affecting the tricuspid valve), or neonatal
toxicity, such as hypothyroidism, nephrogenic diabetes insipidus, or cardiac
arrhythmias. Lithium is contraindicated during the first trimester of pregnancy, and
should be used with caution and close monitoring during the second and third
trimesters.
Olanzapine is an atypical antipsychotic medication that can be used to treat bipolar
disorder, schizophrenia, or major depressive disorder. Olanzapine has a moderate
risk of causing birth defects, such as neural tube defects, or neonatal complications,
such as respiratory distress, hypotonia, or extrapyramidal symptoms.
Risperidone is an atypical antipsychotic medication that can be used to treat
bipolar disorder, schizophrenia, or major depressive disorder. Risperidone has a
moderate risk of causing birth defects, such as cardiac malformations, or neonatal
complications, such as respiratory distress, hypotonia, or extrapyramidal symptoms.
Risperidone can also be secreted into breast milk and cause sedation,
extrapyramidal symptoms, or hyperprolactinemia in the infant, so breastfeeding is
not recommended while taking risperidone.

83. A 40-year-old man is brought to the emergency department by his wife after
having a generalized tonic-clonic seizure that lasted for 2 minutes. He has no prior
history of seizures or neurological disorders. His wife reports that he has been
taking tramadol for chronic back pain for the past year, but he ran out of his
prescription 3 days ago and could not get a refill. On physical examination, he is
alert and oriented, but he has tremors, diaphoresis, and dilated pupils. His vital
signs are blood pressure 160/100 mmHg, pulse 120 beats/min, respirations 24
breaths/min, and temperature 37.2°C. Which of the following is the most
appropriate next step in the management of this patient?
A. Investigate for substance abuse
B. Brain imaging
C. Intravenous lorazepam
D. Intravenous naloxone

Answer: A. Investigate for substance abuse


Explanation: He has signs and symptoms of tramadol withdrawal, which can cause
seizures and autonomic instability. Tramadol is an opioid analgesic that can cause
dependence and abuse. He needs substance abuse evaluation and treatment.

84. A 60-year-old woman is admitted to the hospital with sepsis due to a urinary
tract infection. She has a history of chronic liver disease and diabetes mellitus. She
has been taking paracetamol for fever and pain. She develops oliguria and her
serum blood urea nitrogen (BUN) and creatinine levels are elevated. Her urine
output is less than 0.5 mL/kg/h. Her urine specific gravity is 1.030 and her urine
osmolality is 500 mOsm/kg. Her serum sodium level is 140 mEq/L and her serum
osmolality is 290 mOsm/kg. Which of the following is the most likely diagnosis?
A. Prerenal azotemia
B. Acute tubular necrosis
C. Acute interstitial nephritis
D. Hepatorenal syndrome

Answer: A. Prerenal azotemia


Explanation:
She has signs and symptoms of prerenal azotemia, which is a decrease in renal
perfusion due to hypovolemia, hypotension, or decreased cardiac output. Sepsis
can cause prerenal azotemia by causing vasodilation and hypotension. Prerenal
azotemia is characterized by oliguria, high urine specific gravity and osmolality,
low urine sodium level, and high BUN/creatinine ratio.

85. A 30-year-old man comes to the clinic because of fatigue, headache, and
dizziness for the past 2 weeks. He works as a painter and is exposed to various
solvents and chemicals. He has no history of smoking, asthma, or allergies. On
physical examination, he has pale conjunctiva, tachycardia, and tachypnea. His
chest auscultation is normal. His complete blood count shows pancytopenia. A
chest radiograph is ordered and shows which of the following findings?

A. Pneumatoceles
B. Pulmonary infiltrates
C. Pleural effusion
D. Mediastinal mass
E. Normal lungs

Answer: B. Pulmonary infiltrates


Explanation:
He has signs and symptoms of benzene toxicity, which can cause bone marrow
suppression, anemia, and leukemia. Benzene is a volatile organic compound that
can be found in paints, solvents, and gasoline. Benzene toxicity can cause
pulmonary edema, inflammation, and hemorrhage, which can appear as pulmonary
infiltrates on CXR.
Pneumatoceles are air-filled cysts in the lung parenchyma, usually caused by
bacterial pneumonia, trauma, or mechanical ventilation. They are not associated
with benzene toxicity.

86. A 20-year-old man is brought to the emergency department after being


electrocuted by a high-voltage wire. He has circumferential burns on both hands
and complains of severe pain. His vital signs are blood pressure 150/90 mmHg,
pulse 110 beats/min, respirations 20 breaths/min, and temperature 37°C. Which of
the following is the most likely complication of his burns?
A. Compartment syndrome
B. Hypovolemic shock
C. Infection
D. Myocardial infarction

Answer: A. Compartment syndrome


Explanation:
Compartment syndrome is a condition in which increased pressure within a closed
fascial space compromises the blood supply and function of the tissues within that
space. It can be caused by trauma, burns, fractures, or ischemia. Circumferential
burns can cause compartment syndrome by creating a tight band around the
affected limb, restricting the blood flow and causing edema, pain, and ischemia.
Compartment syndrome can lead to muscle necrosis, nerve damage, and limb loss
if not treated promptly. The treatment of compartment syndrome is surgical
fasciotomy, which involves cutting the fascia to relieve the pressure and restore the
blood flow.
87. A 35-year-old woman is brought to the emergency department by her husband
because of altered mental status and respiratory depression. He says that she has
been using heroin for the past 5 years and that she injected herself with an
unknown amount of the drug an hour ago. On physical examination, she is
unresponsive, her pupils are pinpoint, and her respiratory rate is 6 breaths/min. Her
blood pressure is 90/60 mmHg, pulse is 50 beats/min, and oxygen saturation is
85% on room air. Which of the following is the most appropriate initial
management for this patient?
A. Methadone
B. Naproxen
C. Naloxone
D. Naltrexone

Answer: C. Naloxone
Explanation:
Naloxone is an opioid antagonist that can reverse the effects of opioid overdose,
such as respiratory depression, sedation, and miosis. Naloxone can be administered
intravenously, intramuscularly, subcutaneously, or intranasally, and has a rapid
onset of action. Naloxone can restore the consciousness and respiration of the
patient, but it may also precipitate withdrawal symptoms, such as agitation, pain,
nausea, and vomiting
Methadone is a synthetic opioid agonist that can be used to treat opioid dependence,
as it can prevent withdrawal symptoms and reduce cravings. Methadone is not
indicated for the treatment of acute opioid overdose, as it can worsen the condition
and cause death.
Naproxen is a nonsteroidal anti-inflammatory drug (NSAID) that can be used to
treat pain, inflammation, and fever. Naproxen has no effect on opioid receptors,
and it is not useful for the treatment of opioid overdose or withdrawal.
Naltrexone is an opioid antagonist that can be used to treat opioid dependence, as it
can block the effects of opioids and reduce the reward and reinforcement of drug
use. Naltrexone is not indicated for the treatment of acute opioid overdose, as it has
a slow onset of action and may not be effective in reversing the respiratory
depression.
88. A 45-year-old man with a history of chronic alcohol abuse is admitted to the
hospital for severe abdominal pain. He reports that he stopped drinking alcohol two
days ago after experiencing nausea and vomiting. On physical examination, he is
restless, agitated, and diaphoretic. His blood pressure is 180/100 mm Hg, pulse is
120/min, and temperature is 38.2°C (100.8°F). He has tremors in his hands and
tongue. Which of the following is the most appropriate pharmacologic treatment
for this patient?
A. Thiamine
B. Diazepam
C. Disulfiram
D. Naltrexone

Answer: B. Diazepam
Explanation:
The patient is experiencing alcohol withdrawal syndrome, which is characterized
by autonomic hyperactivity, tremors, anxiety, insomnia, and seizures. The most
appropriate pharmacologic treatment is benzodiazepines, such as diazepam, which
reduce the risk of seizures and delirium tremens. Areke is a traditional alcoholic
beverage in Ethiopia, which would worsen the patient's condition. Disulfiram is an
aversive agent that inhibits aldehyde dehydrogenase and causes unpleasant
reactions when alcohol is consumed. It is used for relapse prevention, not acute
withdrawal. Naltrexone is an opioid antagonist that reduces the rewarding effects
of alcohol. It is also used for relapse prevention, not acute withdrawal. Thiamine is
a vitamin that prevents Wernicke-Korsakoff syndrome, a neurologic complication
of chronic alcohol abuse. It should be given to all patients with alcohol use
disorder, but it does not treat withdrawal symptoms.

89. A 25-year-old woman is brought to the emergency department after being


involved in a motor vehicle collision. She has multiple lacerations and abrasions on
her face and chest. She is conscious and alert, but complains of severe pain in her
right lower leg. On examination, her right leg is pale, cold, and pulseless. She has a
large open wound on her right thigh with active bleeding. Which of the following
is the first action that should be taken to manage this patient?
a. Pressure dressing
b. Tourniquet
c. Intravenous fluids
d. Blood transfusion

Answer: B. Tourniquet
Explanation:
The patient has signs of acute limb ischemia and hemorrhagic shock due to
traumatic injury. The first action that should be taken is to apply a tourniquet above
the wound to stop the bleeding and prevent further blood loss. A pressure dressing
may not be sufficient to control the bleeding from a large open wound. Intravenous
fluids and blood transfusion may be needed to restore the patient's intravascular
volume and oxygen-carrying capacity, but they should be done after the bleeding is
controlled. Heparin is an anticoagulant that may worsen the bleeding and is not
indicated in this situation.

90. A 32-year-old woman is brought to the emergency department after being hit
by a car while crossing the street. She is conscious and alert, but has severe neck
pain and numbness in her arms and legs. She has a large bruise on the right side of
her neck, with swelling and tenderness. She also has difficulty breathing and
speaking. Which of the following is the first action that should be taken to manage
this patient?
A. Airway
B. Breathing
C. Circulation
D. Neck collar

Answer: D. Neck collar


Explanation:
The patient has a blunt neck injury, which can cause cervical spine fracture, spinal
cord injury, vascular injury, or tracheal injury. The first action that should be taken
is to apply a neck collar to immobilize the neck and prevent further damage to the
spinal cord or the vessels. The neck collar should not be removed until the patient
is evaluated by radiography or computed tomography. The other steps of the basic
life support (BLS) algorithm (airway, breathing, circulation) should be done after
the neck collar is applied. A spinal board is used to transport the patient, but it does
not provide adequate immobilization of the neck.

91. A 60-year-old man with a history of type 2 diabetes mellitus presents to the
emergency department with confusion, sweating, and palpitations. He reports that
he skipped breakfast and took his usual dose of insulin. His blood glucose level is
40 mg/dL. He is given an intravenous infusion of 50% dextrose and his symptoms
improve. Which of the following hormones is most likely responsible for his
neuroglycopenic symptoms?
A. Insulin
B. Glucagon
C. Cortisol
D. Epinephrine

Answer: A. Insulin
Explanation:
Insulin is the hormone that lowers blood glucose levels by stimulating glucose
uptake and utilization by peripheral tissues, especially skeletal muscle and adipose
tissue. Excessive insulin action, either due to overdose or inadequate carbohydrate
intake, can cause hypoglycemia, which is defined as a blood glucose level below
70 mg/dL. Hypoglycemia can cause neuroglycopenic symptoms, such as confusion,
lethargy, seizures, or coma, due to insufficient glucose supply to the brain. The
other hormones are counter-regulatory hormones that increase blood glucose levels
by stimulating glycogenolysis, gluconeogenesis, lipolysis, and ketogenesis. They
are released in response to hypoglycemia to restore normoglycemia and prevent
neuroglycopenic symptoms.
92. A 20-year-old woman comes to the clinic for removal of stitches on her face.
She had a laceration on her left cheek that was repaired with sutures 10 days ago.
The wound is well-healed and there is no sign of infection. Which of the following
is the most appropriate time interval for removal of facial stitches?
A. 3 days
B. 5 days
C. 7 days
D. 14 days

Answer: B. 5 days
Explanation:
The optimal time interval for removal of stitches depends on the location and type
of the wound, the type and size of the suture material, and the patient's healing
capacity. In general, facial stitches should be removed within 5 days to minimize
scarring and prevent suture marks. Other areas of the body, such as the trunk,
extremities, or scalp, may require longer intervals, ranging from 7 to 14 days.
Removing stitches too early may increase the risk of wound dehiscence, while
leaving them too long may increase the risk of infection or foreign body reaction.
93. A 5-year-old boy is brought to the emergency department after being
submerged in a freshwater lake for 10 minutes. He is cyanotic, PR 68, and apneic.
What is the most appropriate initial management for this patient?
A. Chest compressions
B. Mouth to mouth
C. Intubation and mechanical ventilation
D. Passive rewarming

Answer: B. Mouth to mouth


Explanation:
The most important initial step in managing a near drowning victim is to establish
an airway and provide adequate ventilation. Mouth to mouth is the simplest and
most effective method of providing rescue breaths in the field. Chest compressions
are only indicated if there is no pulse. Intubation and mechanical ventilation may
be required later, but are not feasible in the field. Rewarming is also important, but
secondary to ventilation.

94. A 40-year-old man is brought to the emergency department after being exposed
to carbon monoxide (CO. in a house fire. He is conscious and alert, but complains
of headache, nausea, and dizziness. His pulse oximetry shows an oxygen saturation
of 98% on room air. What is the best parameter to monitor her response to
treatment?
A. Pulse oximetry
B. Arterial blood gas
C. Carboxyhemoglobin level
D. Methemoglobin level

Answer: C. Carboxyhemoglobin level

Explanation:
The best parameter to monitor the response to treatment of carbon monoxide (CO)
poisoning is the carboxyhemoglobin (HbCO) level, which reflects the amount of
CO bound to hemoglobin. The treatment of CO poisoning is hyperbaric oxygen
therapy, which displaces CO from hemoglobin and reduces tissue hypoxia. SpO2 is
not a reliable indicator of oxygenation in CO poisoning, as it measures the
percentage of hemoglobin that is saturated with either oxygen or CO. Mentation
may improve with treatment, but is not a specific or objective parameter.
PaO2 and PaCO2 are normal in CO poisoning.

95. A term newborn boy is admitted to the neonatal intensive care unit for
respiratory distress and abdominal distension. He was born to a 25-year-old
woman, gravida 2, para 2, who had no prenatal care. The delivery was complicated
by meconium-stained amniotic fluid and fetal bradycardia. The Apgar scores were
3 and 5 at 1 and 5 minutes, respectively. The newborn received positive pressure
ventilation and endotracheal suctioning at birth. On examination, he has tachypnea,
intercostal retractions, and decreased breath sounds on the right side. His abdomen
is distended and tympanic. A plain abdominal radiograph shows a dilated colon
filled with meconium and air-fluid levels. Which of the following is the most likely
diagnosis?
A. Congenital diaphragmatic hernia
B. Meconium ileus
C. Hirschsprung disease
D. Meconium aspiration syndrome

Answer: B. Meconium ileus


Explanation:
Meconium ileus presents with abdominal distension, bilious vomiting, and failure
to pass meconium within the first 48 hours of life. The diagnosis is confirmed by
abdominal radiograph, which shows a dilated colon filled with meconium and air-
fluid levels, and a microcolon distal to the obstruction.
Congenital diaphragmatic hernia is a defect in the diaphragm that allows the
abdominal organs to herniate into the thoracic cavity, causing pulmonary
hypoplasia and respiratory distress. It presents with cyanosis, scaphoid abdomen,
and bowel sounds in the chest. The diagnosis is confirmed by chest radiograph,
which shows a mediastinal shift and air-filled loops of bowel in the thorax.
Hirschsprung disease is a congenital absence of ganglion cells in the distal colon,
causing functional obstruction and failure of relaxation of the internal anal
sphincter. It presents with abdominal distension, constipation, and failure to pass
meconium within the first 48 hours of life. The diagnosis is confirmed by rectal
biopsy, which shows absence of ganglion cells and hypertrophy of nerve fibers.
96. A first day neonate is delivered by cesarean section because of fetal distress.
The neonate has tachypnea, grunting, and intercostal retractions. The chest x-ray
shows hyperinflation and fluid in the fissures. What is the most likely diagnosis?
A. Transient tachypnea of the newborn
B. Meconium aspiration syndrome
C. Hyaline membrane disease
D. Pneumonia

Answer: A. Transient tachypnea of the newborn


Explanation:
TTN is a common condition in neonates delivered by cesarean section, especially
without labor. It is caused by delayed clearance of fetal lung fluid. The symptoms
usually resolve within 48 hours.
MAS is associated with meconium-stained amniotic fluid, fetal distress, and coarse
crackles on auscultation.
HMD is more common in preterm infants and is characterized by surfactant
deficiency, atelectasis, and ground-glass appearance on chest x-ray.

97. A neonate is born at 36 weeks of gestation with respiratory distress and


cyanosis. The chest x-ray shows a mediastinal shift to the right, absence of the left
hemidiaphragm, and bowel loops in the left hemithorax. What is the most likely
diagnosis?
A. Congenital cystic adenomatoid malformation
B. Congenital diaphragmatic hernia
C. Pulmonary sequestration
D. Bronchopulmonary dysplasia (

Answer: B. Congenital diaphragmatic hernia


Explanation:
CDH is a defect in the diaphragm that allows abdominal organs to herniate into the
chest cavity, compressing the lung and displacing the heart. It is usually diagnosed
prenatally or shortly after birth.
CCAM is a hamartomatous lung lesion that may cause respiratory distress, but it
does not cause mediastinal shift or bowel loops in the chest.
Pulmonary sequestration is a non-functioning lung tissue that receives blood
supply from the systemic circulation, but it is usually asymptomatic at birth.
BPD is a chronic lung disease that occurs in preterm infants who received
mechanical ventilation and oxygen therapy.

98. A neonate is born at home and brought to the hospital on the second day of life
because of poor feeding, lethargy, and fever. The neonate has generalized muscle
rigidity, opisthotonus, and trismus. The mother did not receive any antenatal care
or immunizations. What is the most likely diagnosis?
A. Congenital syphilis
B. Neonatal tetanus
C. Hemorrhagic disease of the newborn
D. Neonatal sepsis

Answer: B. Neonatal tetanus


Explanation:
Neonatal tetanus is a fatal disease that is caused by Clostridium tetani infection of
the umbilical stump. It is preventable by maternal immunization with tetanus
toxoid. The clinical features include spasms, rigidity, and lockjaw.
Hemorrhagic disease of the newborn is a bleeding disorder that results from
vitamin K deficiency. It may present with ecchymoses, hematomas, or bleeding
from the umbilicus, but it does not cause muscle spasms.
Neonatal sepsis is a systemic infection that may cause fever, poor feeding, lethargy,
and respiratory distress, but it does not cause rigidity or trismus.
Congenital syphilis is a multisystem infection that may cause rash, snuffles,
hepatosplenomegaly, and skeletal abnormalities, but it does not cause muscle
spasms.

99. A neonate is born at term to a mother with blood group O positive. The neonate
develops jaundice on the second day of life, which progresses to the level of the
sole. The neonate is otherwise well and feeding normally. The direct Coombs test
is negative. What is the most likely cause of the jaundice?
A. Blood group incompatibility
B. Rh incompatibility
C. Breastfeeding jaundice
D. Biliary atresia

Answer: C. Breastfeeding jaundice


Explanation:
Breastfeeding jaundice is a common cause of physiological jaundice in the first
week of life. It is due to suboptimal intake of breast milk, which leads to
dehydration, increased enterohepatic circulation of bilirubin, and decreased stool
output. It usually resolves with adequate breastfeeding and hydration. Blood group
incompatibility and Rh incompatibility are causes of hemolytic jaundice, which are
associated with positive Coombs test, anemia, and splenomegaly. Biliary atresia is
a cause of obstructive jaundice, which is characterized by acholic stools, dark urine,
and hepatomegaly.

100. A preterm neonate is born at 28 weeks of gestation and weighs 1.2 kg. The
neonate is admitted to the neonatal intensive care unit and receives parenteral
nutrition and surfactant therapy. What is the most important nutritional supplement
for this neonate?
A. Vitamin d
B. Iron
C. Vitamin k
D. Folic acid

Answer: A. Vitamin D
Explanation:
Vitamin D is essential for calcium and phosphorus metabolism and bone
mineralization. Preterm infants are at risk of vitamin D deficiency and rickets due
to low maternal and fetal stores, decreased exposure to sunlight, and impaired renal
function. Vitamin D supplementation is recommended for all preterm infants. Iron
is important for erythropoiesis and prevention of anemia, but it is not as critical as
vitamin D in the immediate neonatal period.
Vitamin K is given to all newborns at birth to prevent hemorrhagic disease, but it is
not a supplement that needs to be continued.
Folic acid is important for DNA synthesis and prevention of neural tube defects,
but it is not a supplement that needs to be given to preterm infants.

You might also like